Qqad - 2008

Download as pdf or txt
Download as pdf or txt
You are on page 1of 71

Solution to Quantitative Question # 001 (solved first by implex, nbangalorekar,

warrior)
--------------------------------------------------------

Let the sum S = 20 of four natural numbers a, b, c, d be such that a(a+1) + b(b+1) +
c(c+1) + d(d+1) = 312. Which among the a, b, c, d is/are uniquely determinable ?

(1) none if a = b (2) Atleast 2 if a ≠b (3) All if a > b (4) All of the
foregoing (5) Exactly 2 of the foregoing

Solution:

Given a(a+1) + b(b+1) + c(c+1) + d(d+1) = 312 => (a-1)^2 + (b-1)^2 + (c-1)^2 + (d-1)^2 = 312 -
3S + 4 = 256. Let a-1 = A, b-1 = B, c-1 = C, d-1 = D => we have non-negative integers A, B, C, D
such that A^2 + B^2 + C^2 + D^2 = (A+B+C+D)^2 (the LHS <= RHS always) and can only be
true if all but one number is zero => three among a, b, c, d are 1 and fourth number is 17.

Thus (3) holds true - (2) can guarantee the values for c and d. For a=b we have exactly 2
determinable values - thus (1) is not true.

=> Choice (5) is the right answer

Solution to Quantitative Question # 002 (solved correctly by jha16june)


--------------------------------------------------------

Consider a string of n 7s, 7777....77, into which + signs are inserted to produce an
arithmetic expression. For example, 7 + 777 + 7 = 791 could be obtained from five 7s
in this way. For how many values of n is it possible to insert + signs so that the
resulting expression has value 7000?

(1) 105 (2) 106 (3) 108 (4) 109 (5) 111

Solution:

Let the arithmetic expression on the left side that has + signs in the LHS has p 7s, q 77s and q 777s
=> we have to find n such that p+2q+3r = n and 7p + 77q + 777r = 7000 => p + 11q + 111r = 1000.
Solving we get 9(q+12r) = 1000-n; now n can vary from 1000 to 1 out of which n = 1, 10, 19, 37 are
excluded as p <0 for these values => total such n are [999/9 + 1] - 4 = 108.

=> Choice (3) is the right answer

Solution to Quantitative Question # 003 (solved correctly by implex)


--------------------------------------------------------

Let f(x) be a function such that f(x).f(y) - f(xy) = 3(x+y+2). Then f(4) (equals)

(1) can not be determined (2) 7 (3) -8 (4) either 7 or -8 (5) none of
these

Solution:

Given f(x).f(y) - f(xy) = 3(x+y+2). Put x=y=0 => and let f(0) = p => p^2 - p - 6 = 0 => p = 3, -2 =
f(0).
In f(x).f(y) - f(xy) = 3(x+y+2), put y = 0 => for f(0) = 3 we get f(x) = x+3, for f(0)
= -2 we get f(x) = -(3x+4)/2.

Check for both f(x) = x+3 and -(3x+4)/2 on putting in the original functional
equation f(x).f(y) - f(xy) = 3(x+y+2). We see f(x) = -(3x+4)/2 fails and f(x) = x+3
holds true.
Ö Choice (2) is the right answer

Solution to Quantitative Question # 004 (solved correctly by implex, slam, kavita_iet,


OperationBLACKI, jigar_er_civil, thebornattitude, mejogi, srk_soumya, abhishekag_747, arvindva,
nishisoni, maverick_srikan, sabsebadapaagal, kharesaurabh84)
--------------------------------------------------------

N students are seated at desks in an m x n array, where m, n >= 3. Each student


shakes hands with the students who are adjacent horizontally, vertically or
diagonally. If there are 81 handshakes, what is N?

(1) 28 (2) 27 (3) 30 (4) 25 (5) 24

Solution:

Students in the corner shake hands 3 times, those on the sides 5 times and those in the middle 8
times. So the total number of handshakes is (4•3 + (2m-4+2n-4)5 + (m-2)(n-2)8)/2 = (12 + 10m
+ 10n - 40 + 8mn - 16m - 16n + 32)/2 =
(16mn - 12m - 12n + 8)/4 = (4m - 3)(4n - 3)/4 - 1/4 = 81, so (4m-3)(4n-3) = 325 = 13.25.
hence N = mn = 4•7 = 28.

=> Choice (1) is the right answer

Solution to Quantitative Question # 005


--------------------------------------------------------

A particle moves around a circle (once) such that its displacement from the initial
point in given time t is t(6-t) meters where t is the time in seconds after the start.
The time in which it completes one-sixth of the distance is

(1) 0.60 s (2) 0.88 s (3) 1 s (4) 1.12 s (5) none of these

Solution:

The displacement from the starting position can be max when the particle (P) is
at diametrically opposite position of S => when t(6-t) is max then the value of
t(6-t) = 2R where R is the radius of the circle => t = 3 and R = 9/2.

Now, when the particles covers 1/6th of the distance => the angle subtended by
SP at the center of the circle is 360/6 = 60 degrees => length of SP is R = 9/2 =
t(6-t) => t = 0.88
=> Choice (2) is the right answer

Solution to Quantitative Question # 006


--------------------------------------------------------
In a soccer tournament n teams play against one another exactly once. The win
fetches 3 points, draw 1 each and loss 0. After all the matches were played, it was
noticed that the top team had unique number of maximum points and unique least
number of wins. What can be the minimum possible value of n?

(1) 5 (2) 6 (3) 7 (4) 8 (5) none of these

Solution:

Total number of matches played =n(n-1)/2


Total matches per team =n-1
For getting the least number of teams,
the difference in number of wins will be minimum ..i.e 1
Assume All other teams (except winning team) have same number of wins.
If some team other Y than the top team X had 2 more wins than X => Y had 6 points more than X
from the wins => X would need 7 more draws than X => n will be > 9 here.
Assuming K wins for winning team and n-1-k draws and k+1 wins for other teams
we have ,
Total matches(based on wins and draws =k + (n-1-k) + (n-1) (k+1)
=(n-1) (k+2)
This should be equal to n(n-1)/2 (total matches played btw n teams )
==> k= (n-4)/2------------------------------------(A)
So from this we know n is even,
now the points that the winning team gets =3k +n-1-k =2k+ n-1
points other team gets (maximum) = 3(k+1) + 1=3k+4
Given that 2k+n -1 > 3k+4 ==> n > k+5
substituting k=(n-4)/2
we get 2n > n+6 or n > 6 -------------------------(B)
==> Since n is even, 8 is the answer

Example for n = 8 as possibility.

Name the 8 teams X, A, B, C ,D, E, F, and G

X defeats A and B, and draws against C ,D, E, F, and G for 2 wins and 11 points.
A defeats B, D, and E for 3 wins and 9 points.
B defeats C, D, and F for 3 wins and 9 points.
C draws against X and defeats A, E, and G for 3 wins and 10 points.
D draws against X and defeats C, E, and G for 3 wins and 10 points.
E draws against X and defeats B, C, and F for 3 wins and 10 points.
F draws against X and defeats A, D, and G for 3 wins and 10 points.
G draws against X and defeats A, B, and E for 3 wins and 10 points.

Alternate solution:

Let X be the desired team. Now let n denote the number of soccer teams which participated in this
tournament, so there are n(n-1)/2 matches played among these teams, now note that each game
gives at least 2 points in total, so sum of the points of all teams is greater than n(n-1) so the
average is at least (n-1).
Note that X has played (n-1) games, and also we know that X has the most total points, so the
total points of X is more than (n-1) so X must have won at least one game.
X has won the fewest number of games, so every other team must have won at least 2 games =>
every other team has at least 6 points => X must draw at least 4 games in order to have the most
total points, but note that if team Y had a draw with team X then the total points of Y would be at
least 7, hence X must have drawn at least 5 games => n >= 7.
We show that n > 7:
if n = 7 then let S denote the set of all teams, X has won a single match and has drew matches, so
the total points of X is equal to 8, now note that every other team has won exactly two games, and
drew at most one game => every other team has lost at least games, so there are at least 18 looses
in => there must be at least 18 wins in ,but we know that the number of wins in S is equal to 1+
6*2 = 13,which is a contradiction.

Example for n = 8 as possibility.

Name the 8 teams X, A, B, C ,D, E, F, and G

X defeats A and B, and draws against C ,D, E, F, and G for 2 wins and 11 points.
A defeats B, D, and E for 3 wins and 9 points.
B defeats C, D, and F for 3 wins and 9 points.
C draws against X and defeats A, E, and G for 3 wins and 10 points.
D draws against X and defeats C, E, and G for 3 wins and 10 points.
E draws against X and defeats B, C, and F for 3 wins and 10 points.
F draws against X and defeats A, D, and G for 3 wins and 10 points.
G draws against X and defeats A, B, and E for 3 wins and 10 points.

=> Choice (4) is the right answer

Solution to Quantitative Question # 007


--------------------------------------------------------

In a quadrilateral ABCD sides AB and CD are equal with <A = 150˚, <B = 44˚, and <C
= 72˚. Perpendicular bisector of the segment AD meets meets the sides BC at point
X. Then m(<AXD) is

(1) 42˚ (2) 58˚ (3) 64˚ (4) 78˚ (5) none of these

Solution:

Let P be a point on BC such that AP = AB => <BAP = 180˚ – 2*44˚ = 92˚


Let Q be a point on BC such that DQ = CD => <CDQ = 180˚ – 2*72˚ = 36˚

Let E be the intersection point of AP, DQ.

We have <EAD = 150˚ -92˚ = 58˚ and <EDA = 94˚ – 36˚ = 58˚

So the triangle EAD is isosceles => EA = ED. Also AB = CD => AP = DQ

We'll prove that E lies on BC.

Assume that E doesn’t lie on BC

The line BC bisects the plane into two half-planes


If E is on the same half-plane with A, D then EP = AP - EA and EQ = DQ - ED
If E is on the other half-plane with A, D then EP = EA - AP
and EQ = ED - DQ

At any case, from we get EP = EQ which yields that EPQ is isosceles.


But this is not possible because the lines AP and DQ make different angles with line BC.

We got in a contradiction because we assumed that E does not lie on BC. So the only possible case
is that E lies on BC => <AXD = 64˚
Alternate solution:

Let XQ be the perpendicular bisector of AD such that Q lies on AD. Let <AXD = 2x => <AXQ =
<DXQ = x.

=> <BAX = 60˚+x and <XDC = 4˚+x => <AXB = 76˚ -x and < DXC = 104˚-x. By sine rule DC/DX
= AB/AX = sin(104-x)˚/sin72˚ = sin(76-x)˚/sin44˚ => x = 32˚

=> Choice (3) is the right answer

Solution to Quantitative Question # 008


--------------------------------------------------------

Let S be the set of first 14 natural numbers. A special subset of S is a subset S' which
satisfies the following three properties

a) S' has exactly 8 elements


b) If x belonging to S is even, then x is in S' if and only if x/2 is in S'
c) If y belonging to S is odd, then y is in S' if and only if (y+15)/2 is in S'

Let X denotes elements of S that cannot be the part of special subset. Then n(X) (i.e.
number of elements in X) equals

(1)2 (2) 3 (3) 5 (4) 6 (5) none of these

Solution:

This is a problem that was raw and required playing with numbers.

The sets satisfying conditions (b) and (c) are {1, 2, 4, 8}, {3, 6, 9, 12}, {7, 11, 13, 14} and {5, 10}.

By condition (a) we can have 3 combinations of union of 2 sets each from above 3 sets having 4
elements each => set {5, 10} is excluded.

=> Choice (1) is the right answer

Solution to Quantitative Question # 009


--------------------------------------------------------

A shokeeper sells 2 different brands of an article, one for Rs 10 and other for Rs 12
each. One day he left the shop in the hands of his management graduate son who
confused the two brands and sold them at each other's price. Due to this, the
shopkeeper lost Rs 40 which amounted to 10% fall in his expected profit. If
shopkeeper's expected profit was one among 10%, 15%, 20%, or 25% then

(1) the number of cheaper brand sold at higher cost was 60


(2) the expected profit was 20%
(3) if the profit per article of expensive brand is 40 paise more than that of cheaper
brand => the least number of articles of expensive brand he had to sell to recover
his loss would be 34
(4) at least two of the foregoing
(5) none of the foregoing

Solution:
For first article SP = 10, sold "a" items, profit per item = (Pa), sold at 12 Rs per item

For Second article, SP = 12, sold "b" items, profit per item = (Pb), sold at 10 Rs per item

expected profit = a(Pa) + b(Pb)


actual profit = a(Pa +2) + b(Pb -2)

expected - actual = 40 => b = a+20


also, 40 is 10% of expected profit => expected profit = 400 rs.
now, expected profit = total cost * profit percentage/100
total cost = 10a + 12b - 400 = 22a + 240 -400 = 22a -160
if profit% = 10, 22a = 4160 [ a is not an integer]
if profit% = 15, 22a = 400 * 100/15 + 160 [ a is not an integer]
if profit% = 20, 22a = 2160 [ a is not an integer]
if profit% = 25, 22a = 1760
(a has to be a positive integer since it is the number of items sold, so only profit = 25% would be
possible).
expected profit = 25%

also,a = 1760/22 = 80
=> b = 100

For option (3)


80(Pa) + 100 (Pb) = 40 (expected profit)
put Pb = Pa + 0.40
to get Pa = 2, and Pb = 2.40

=> to make up the lost profit of Rs 40, he needs to sell 40/2.4 = 16.66 => 17 items of the higher
price, which is less than 34.

=> Choice (5) is the right answer

Solution to Quantitative Question # 010


--------------------------------------------------------

The total ordered pair of positive integers (p, q) such that the roots of equations x^2
-px + p + q - 3 = 0 and x^2 -qx + p + q - 3 = 0 are also positive integers are

(1) 2 (2) 3 (3) 4 (4) 6 (5) none of these

Solution:

Let a, b be the roots of 1st equation and c, d be the roots of 2nd equation

=> ab = cd = p+q-3 => (ab+cd)/2 = p+q-3 = (a+b) + (c+d) - 3


=> ab + cd = 2(a + b + c + d) - 6 => (a-2)(b-2) + (c-2)(d-2) = 2

Now, given that a, b, c, d are all integers such that ab = cd => 1.1 + 1.1 = 2, -1.-1 + -1.-1 = 2, 1.2 +
0.x = 2, 0.y + 1.2 = 2 => we get (a, b, c, d) as (1, 1, 1, 1), (3, 3, 3, 3) and (2, 6, 3, 4)

Thus, (p, q) = (2, 2), (6, 6), (7, 8) -> these are unordered pairs, an extra permutation (8, 7) of (7,
8) will
give 4 ordered pairs.

=> Choice (3) is the right answer


Solution to Quantitative Question # 011
--------------------------------------------------------

Let ABC be a triangle and D and E be internal points on AB and AC respectively. The
segment s joining AE and CD divide the triangle into 4 regions. If all the 4 regions
have equal area then the triangle ABC is

(1) not possible (2) obtuse-angled (3) equilateral (4) isosceles with
< A = 36˚ (5) none of these

Solution:

Let F be the point of intersection of AE and CD. Given area of each of BFC = CFE = EFDA = DFB
= A => Area of ABE = Area of EBC => BE is median. Similarly, CD is a median => F is the
centeroid. But Area CFE = Area BFC shows CF is median in triangle ECB => F divides BE in 1:1
which is a contradiction as centeroid divideds the median in the ratio 1:2.

=> Choice (1) is the right answer

Solution to Quantitative Question # 012


--------------------------------------------------------

How many eight letter words exist that are composed of Xs and Ys, and which
contain neither three consecutive Xs nor three consecutive Ys?

(1) 74 (2) 66 (3) 76 (4) 68 (5) none of these

Solution:

Let F(n) denote the number of n-letter words consisting of Xs and Ys and not containing three
consecutive identical letters. Thus F(1) = 2 since the one-letter words X and Y meet the no-triple-
repeat condition. Since all four two-letter possibilities XX, XY, YX and YY meet the condition, we
have F(2) = 4.

Assuming that n >= 3, how can we construct an n-letter word with no triple repeats? One way
would be to start with an (n 2)-letter word w and to append either XX if w ends in Y, or YY if w
ends in X. Since there are F(n 2) possibilities for w, this allows us to construct F(n 2) words
of length n, and these are all of the n-letter words with no triple repeats and having a repeated
letter at the end. Similarly, we can start with an (n 1)-letter word w and append either X if w
ends in X, or Y if w ends in X.

This yields F(n 1) acceptable words, and these are all the acceptable n-letter words whose last
two letters are different. We thus have a total of F(n 2) + F(n 1) n-letter words with no triple
repeats, and thus F(n) = F(n 2) + F(n 1). Now F(1) = 2 and F(2) = 4, so F(3) = 2 + 4 = 6, F(4)
= 4 + 6 = 10, F(5) = 6 + 10 = 16,F(6) = 10 + 16 = 26 , F(7) = 16 + 26 = 42, F(8 ) = 26 + 42 = 68.

=> Choice (4) is the right answer

Solution to Quantitative Question # 013


--------------------------------------------------------

Let x and y be positive real numbers such that x^3 + y^3 = 4x^2. A is the maximum
value of x + y.
Let a, b, c be real such that a+b+c = 5 and ab + bc + ca = 3. B is the largest possible
value of c.

Then A + B lies in the range

(1) [7, 8) (2) [8, 9) (3) [9, 10) (4) [10, 11) (5) none of these

Solution:

Let u take the two problems of finding A and B one by one.

Put y = xk in x^3 + y^3 = 4x^2 => x^2(x + k^3x) = 4x^2 => x(1+k^3) = 4. We need to
maximize x + xk = x(1+k) = 4(1+k)/(1+k^3) = 4/[(k-1/2)^2 + 3/4)] = 16/3 = A

For the second part, please see that (a-1/3)^2 + (b-1/3)^2 + (c-1/3)^2 = 16 => c can be
maximized when a = b = 1/3 and c = 13/3.

Alternatively: a+b+c = 5 and ab + bc + ca = 3, replace a by (5-b-c) in second equation and form


a quadratic in b
=> b^2 + (c-5)b + (c^2-5c+3) = 0, again for real b Discriminant >0
=> -3c^2+10c+13 >= 0 => c <= 13/3

=> Choice (3) is the right answer

Solution to Quantitative Question # 014


--------------------------------------------------------

Let x be the number of base systems in which the largest 3 digit perfect square in
base 6 can be represented as a 2 digit number. Then x in base 7 is a

(1) odd but not prime (2) prime (3) even and perfect square (4)
even but not perfect square (5) none of these

Solution:

Let ABC be the largest 3 digit perfect square in base 6 (A, B, C <= 5) => in base 10 ABC will be
36A + 6B + C and this number is a perfect square. Since 5*(36 + 6 + 1) = 215 => 36A + 6B + C =
14^2 = 196 => ABC = 524.

Now, 524 is written as a 2 digit number CD in base b => b*C + D = 524 where C, D < b and C is a
positive integer => b ranges from 23 to 524 => x = 502 = 1315 in base 7.

The properties (odd/even/prime/perfect square) of a number N in base b is same as that of


decimal conversion of N => x is even in base 7 also.

=> Choice (4) is the right answer


Solution to Quantitative Question # 015
--------------------------------------------------------

How many (a, b, c) satisfy log(2ab) = loga*logb, log(bc) = logb*logc, log(2ac) =


loga*logc ?

(1) none (2) 1 (3) 2 (4) 4 (5) none of these

Solution:

log20 = (loga - 1)*(logb - 1), 1 = (logb - 1)*(logc - 1), log20 = (logc - 1)*(loga - 1)

Multiplying 3 equations and taking square root we get


+/-log20 = (loga - 1)*(logb - 1)*(logc - 1)
=> logc = 1 +/-1, loga = 1 +/-log20, logb = 1 +/-1
Thus, (a, b, c) = (200, 100, 100) or (1/2, 1, 1)

=> Choice (3) is the right answer

Solution to Quantitative Question # 016


--------------------------------------------------------

A PaGal is stationed at C, 60 meters directly west of a CAT located at M. The CAT is


trying to escape running at 7m/s in a direction 30 degrees east of north. The PaGal
an expert in geometry runs at 13m/s in a suitable straight line path that will
intercept the CAT as quickly as possible.
Then which among is following is true?

(1) It takes 15/2 seconds for the PaGaL to catch CAT


(2) If CAT chooses a different direction to escape, irrespective of the direction, all
interception points lie on a circle.
(3) If the CAT is intercepted after running a distance of x meters in a particular
direction and
if the CAT had been intercepted after it had run a distance of y meters in the
opposite
direction then min (x+y) is 14√30
(4) All of the above
(5) Exactly 2 of the above

Solution:

Let's try to draw the figure for this. Assume M as the origin (0, 0) where CAT is initially and let
the
PaGal be at P (-60, 0). Let both of run such that PaGal intercepts the CAT after t seconds T => MT
= 7t and PT = 13t, but given that < PMT = 120 degrees. Using cosine rule in the triangle, we have
cos(120˚) = (PM^2 + MT^2 - PT^2)/(2.PM.PT) = -1/2 = (3600 + 49t^2 - 169t^2)/(2.60.7t) => t
= 15/2.
Let (x,y) be locus of point where the CAT and PaGal meets. Point M is considered to be the origin.
(A) = angle from x-axis by which CAT runs off
(B) = angle from x-axis by which PaGal runs off.
(T) = time at which they CAT is busted.

Equations for CAT : x=7tcos(A) , y=7tsin(A)


Equations for PaGal : x=13tcos(B) - 60 , y=13tsin(B)
Since A,B,T are variables for locus. Eliminating them we get the equation
(x-24.5)^2 + y^2 = 2070.25, which is equation of a circle with centre (24.5,0) and radius =
sqrt(2070.25) = 45.5.

Part (c) is easy to see. In the above equation, substitute x= 0, calculate y, and double it to get
14√30 m.

=> Choice (4) is the right answer

Solution to Quantitative Question # 016


--------------------------------------------------------

Let R(x) be the remainder when x^16 + x^8 + x^6 + x^4 + x^2 + 1 is divided by x^3 -
1. Let D(x) be the divisor (less than degree 4) of x^6 + 4x^3 + 8. Then which among
the following is true?

(1) The sum of the coefficients of R(x) and D(x) is equal


(2) The sum of the absolute value of coefficients of R(x) and D(x) is equal
(3) R(x) > D(x) for all non-positive x
(4) at least 2 of the above
(5) none of the above

Solution:

X^16 + x^8 + x^6 + x^4 + x^2 + 1 = Q(x).(x^3 - 1) + R(x), where R(x) has degree lesser than that
of the quotient. Thus if we put x^3 = 1 in both LHS and RHS we get, x + x^2 + 1 + x + x^2 + 1
= 2(x^2 + x + 1) = R(x)

For D(x), as we know that a^3 + b^3 + c^3 - 3abc = (a+b+c)(a^2 + b^2 + c^2 - ab - bc - ca) x^6 +
4x^3 + 8 has a quadratic factor D(x) as x^2 - 2x + 2 as we can put a = x^2, b = -2x and c = 2.
Want to try factorizing x^6 + 5x^3 + 8?

=> Choice (5) is the right answer

Two kinds of Vodka are mixed in the ratio 1:2 and 2:1 and they are sold fetching the
profit 10% and 20% respectively. If the vodkas are mixed in equal ratio and the
individual profit percent on them are increased by 4/3 and 5/3 times respectively,
then the mixture will fetch the profit of

(a) 18% (b) 20% (c) 21 % (d) 23% (e) Cannot be determined

Let the CP of two vodkas be Rs 100 and Rs 100x and individual profit in Rs on them being A and
B.
=> (A+2B)/3 = 10/100*(100+200x)/3 and (2A+B)/3 = 20/100*(200 + 100x)/3. solving we get A
= (70+20x)/3 and B = (20x-20)/3
=> profit percentages on each is (70+20x)/3 and (20x-20)/3x. When they are increased to 4/3
and 5/3 times respectively and mixed in the ratio 1:1 we get total profit % as
(4/3*100*(70+20x)/3 + 5/3*100x*(20x-20)/3x)/(100+100x) = 100*(20x+20)/(100+100x) = 20
=> choice (b) is the right answer.

Solution to Quantitative Question # 018


--------------------------------------------------------

Two spherical balls lie on the ground touching. If one of the balls has a radius of 8
cm, and the point of contact is 10 cm above the ground, what is the radius of the
other ball?

(1) 18 cm (2) 40/3 cm (3) 25/2 cm (4) 13 cm (5) none of the these

Solution:

This is a simple problem on similar triangles. Please refer pages 99-100 in the discussions thread
for figure and explanation.

=> Choice (2) is the right answer

Solution to Quantitative Question # 019


--------------------------------------------------------

The question is followed by two statements X and Y. Answer each question using the
following instruction:

Choose 1 if the question can be answered by X only


Choose 2 if the question can be answered by Y only
Choose 3 if the question can be answered by either X or Y
Choose 4 if the question can be answered by both X and Y
Choose 5 if the question can be answered by neither X nor Y

The positive integers are such that p < q ≤ r < s < 100, ps = qr and √s - √p ≤ 1. What
is the value of p?

(X) The last digit of s is either 1, 2 or 3


(Y) 50 < p and r < 90

Solution:

Given that s-p > r -q as p < q <= r < s


=> (s-p)^2 > (r-q)^2 => (s-p)^2 + 4sp > (r-q)^2 + 4rq [becuase sp = rq, we add 4sp in LHS and
4rq in RHS]
=> (s+p)^2 > (r+q)^2
=> s+p >= r+q+1 [as all numbers are integers] -> (1)

Suppose √s - √p = 1 (the other possibility is √s - √p < 1 that we will see later)


=> s + p - 2√sp = 1 => s+p = 1 + 2√qr [becuase sp = rq] but (1) tells that √qr >= q + r => r = q [By
AM-GM rule on positive numbers] and p+s = 2q + 1.
Now ps = q^2 => gcd(p, s) = 1 (the explanation is below)
Because if x divides gcd (p, s) and x is prime (or it will be product of two or more primes, but we
assume the base case which covers other case as well), then x would divide q [because p = ax, q =
bx => ps = abx^2 = q^2 and gcd(a, x) = 1 and gcd(b, x) = 1) and thus x dividing 2q+1 [= p+s =
x(a+b)] is a contradiction => each of p and s is a perfect square [gcd(p, s) = 1].

If √s - √p < 1 then p + s < 1 + 2√ps <= 1 + q + r <= p + s which is a contradiction.

=> In all s and p are perfect squares. Now take X -> only possible s is 81 => p = 64
Now take Y, p > 50 => p can be 64 or 81 but if p = 81 then s = 100 (not possible as s < 100) => p
can only be 64. The information on r is required to cross-check if our data in hand is correct and it
indeed is as √64.81 = 72.

=> Choice (3) is the right answer

Solution to Quantitative Question # 020


--------------------------------------------------------

For all integers x, y, f(x, y) is defined as f(x+2, y+1) = f(f(x+1, y), f(x, y)) and f(x+1, 0)
= f(x, 1), then f(f(2, 3), f(2, 2)) =

(1) f(4, 5) (2) f(3, 3) (3) f(3, 4) (4) f(4, 3) (5) none of these

Solution:

f(x+2, y+1) = f(f(x+1, y), f(x, y)) [Given] -> (1)


f(x+1, 0) = f(x, 1) Given -> (2)
put x=x-1 and y=0 in (1) we get
f(x+1,1) = f(f(x,0),f(x-1,0)) = f(f(x-1,1),f(x-2,1)) = f(x,2) -> (3)

also f(x+2,2) = f(f(x+1,1),f(x,1) = f(f(x,2),f(x-1,2)) = f(x+1,3) -> (4) [using 3]


going further we can see that f(x+n, n) = f(x+n-1, n+1) for positive integer n -> (5)
[can be proved using Mathematical Induction]

=> f(f(2,3), f(2,2)) = f(f(3,2),f(2,2) = f(4,3) [using 4]


But f(4, 3) = f(3, 4) [using 5]

=> Choice (3) and (4) are the right answer

Solution to Quantitative Question # 021


--------------------------------------------------------

A point at the intersection of two or more grid lines is called a lattice point. S is a 5 x
5 array of lattice points. How many squares have their vertices in S?

(1) 30 (2) 36 (3) 44 (4) 50 (5) none of these

Solution:

For 3X3 array we have in all 5 squares (when joined vertically or horizontally) such that we have
2^2 squares of side 1X1, and 1 square of size 2X2. But when mid-points of the 2X2 square is
joined, we have another square whose all vertices are on the perimeter of the array of 3X3. Please
note that both the 2X2 square have all the vertices on the perimeter of 3X3 array. Thus, in all we
have 4 + 2 = 6 such squares for 3X3.

For 4X4 array we have 9 squares of size 1X1, 3 squares (whose vertices lie on the perimeter of the
array) of size 3X3 and 4*2 squares of size 2x2 [4 is conventional we all know, we double it as the
midpoints of the edges joining will also form an equal number of squares] => In all 9 + 8 + 3
squares.

Now for our problem in hand 5X5:


We have 16 ((n-1)^2, when array is nXn) squares of size 1X1, 4 (n-1, when array is nXn) squares
(whose vertices lie on the perimeter of the array) of size 3X3, 18 squares (2*(n-2)^2 of size 2X2)
and 12 squares (3*(n-3)^2 of size 3X3) [for 3X3 we have trepled as done in 4X4 logic above]
=> In all 4 + 16 + 18 + 12 = 50 squares.

=> Choice (4) is the right answer

Solution to Quantitative Question # 022


--------------------------------------------------------

Let a sequence S(n) be defined for positive integers n such that S(0) = 1 and S(1) = 1.
If S(n+2) = 2S(n+1) + S(n), and S(n+1)/S(n) approaches a finite number R as n ->
Infinity, then R equals

(1) √2 + 1 (2) 5/2 (3) √5 (4) 3√3/2 (5) none of these

Solution:

Given S(n+2) = 2S(n+1) + S(n); divide both the sides by S(n+1) => S(n+2)/ S(n+1) = 2 +
S(n)/S(n+1).

Also given that S(n+1)/S(n) -> R as n -> Infinity => S(n+2)/S(n+1) -> R as n ->
Infinity

Thus, R = 2 + 1/R, since R > o => R = √2 + 1


=> Choice (1) is the right answer

Solution to Quantitative Question # 023


--------------------------------------------------------

ABCD is a square, point E is inside triangle ACD and point F is inside triangle ACB. <
EAF = <ECF = 45˚. If DE = 3, and BF = 4, then EF equals

(1) 5 (2) 2√3 (3) 7/2 (4) 24/7 (5) none of these

Solution:

Reflect the point B in the lines AF and CF; let the reflections be R and S, respectively. It follows
from the reflection that AR=AB=AD, and by adding up the angles at vertex A we have < EAR = <
DAE. Hence the triangles AER and AED are congruent. Similarly, the triangles CES and CED are
also congruent.

It follows from the congruences that the following angles are equal
< CSF = <CBF,< CSE = < CDE, < ERA = < EDA, < CSE = < CDE

=> FR = FS = FB = 4 and ER = ES = ED = 3 . The triangles FER and FES are congruent because
they have pairwise equal sides. Thus their angles at R and S are equal => the sum of these two
angles equals the sum of the angles of the square ABCD at B and D, => < FRE = < FSE = 90˚.
Hence with the Pythagorean theorem, EF = 5.

Alternative Solution:

Let's use co-ordinate for our problem. Let A (0, 0), B (a, 0), C (a, a), D (0, a) and E = (p, q) and F
= (r, s). Thus, the equations of line AE is qx - py, line AF is sx - ry, line CE is (a-q)x - y(a-p) = a(p-
q), line CF is (a-s)x - y(a-r) = a(r-s).
Let the notation tanI denotes Inverse of tan, as we know tanI (<EAF ) = 1 => tanI(q/p) - tanI(s/r)
= 1 i.e. the angle between line AE and AF -> (1)
Similarly, 1 + tanI[(a-q)/(a-p)] = tanI[(a-s)/(a-r)] -> (2)

From 1 we get qr = sp+pr+qs -> (3) from 2 we get that 2a^2 - 2ar -2qa + pr + qr + qs - ps = 0 ->
(4)

We know that p^2 + (q-a)^2 = 9 and (r-a)^2 + s^2 = 16 and we have to find square root of (p-
r)^2 + (q-s)^2, which is indeed √25 [from (3) and (4)]

=> Choice (1) is the right answer

Solution to Quantitative Question # 024


--------------------------------------------------------

The number of those subsets of {1, 2, 3, 4, 5, 6} such that the equation x+y = 7 has no
solution in it is

(1) 18 (2) 21 (3) 27 (4) 36 (5) none of these

Solution:

Consider 3 pairs (n, 7-n) where n is 1, 2, 3. Thus, for each pair we have 3 possibilities, both aren't
in the set, only n is in the set, only 7-n is in the set => in all 3^3 = 27 possibilities.

=> Choice (3) is the right answer

Solution to Quantitative Question # 025


--------------------------------------------------------

In a test taken by 100 students, 60 cleared cut-off in DI, 44 cleared cut-off in


mathematics, 38 cleared cut-off in English and 27 students cleared cut-off in GK. 20
students cleared cut-off in all 4 sections. How many maximum students could have
failed to clear the cut-off in all four sections?

(1) 38 (2) 41 (3) 47 (4) 50 (5) none of these


Solution:

In all we have 40 fails in DI, 56 fails in quant, 62 fails in VA/RC and 73 fails in GK, and 20 fails in
none.

=> 1 Fails + 2 Fails + 3 Fails + 4 Fails = 80 (students).

At most we can have 40 students who fail all 4 sections [considering this as true]
1 Fails (x) + 2 Fails (y) + 3 Fails (z) = 40
and we still have 0 fails in DI, 16 fails in quant, 22 fails in quant and 33 fails in GK.
=> x + 2y + 3z = 16 + 22 + 33 = 71.

Solving for this, we get 1 solution set (x, y, z) as (1, 24, 15) which is feasible
=> 40 can be the maximum people who have 4 fails.

=> Choice (5) is the right answer

Solution to Quantitative Question # 026


--------------------------------------------------------

Let ABCD be a rectangle with AB = a, and BC = b. Suppose x is the length of the


radius of the circle passing through A and B and touching CD, and y be the length of
the circle passing through B and C and touching AD. If x + y ≥ k.(a+b) for all a and
b, then k =

(1) √3/2 (2) 5/8 (3) 1/√2 (4) 1/2 (5) none of these

Solution:

Let O be the centre of the circle which touches CD and passes through the points A and B, E be the
point of tangency and F be the point at which OE meets AB. Since OE is perpendicular to DC and
AB || DC => OE is perpendicular to AB. Also, OF = b-x, and FB = a/2. In triangle OFB, OB^2 =
OF^2 + FB^2 => x^2 = (b-x)^2 + (a/2)^2 => x = b/2 + a^2/8b. Silmilarly, y = a/2 + b^2/8a

=> x+y = (a+b)/2 + (a^3 + b^3)/8ab >= (a+b)/2 + (a+b)/8

=> Choice (2) is the right answer

Solution to Quantitative Question # 027


--------------------------------------------------------

Vineet has Rs 600 with him. Each day he buys either beer for Rs 100 or vodka for Rs
200 or whisky for Rs 200. In how many ways can Vineet spend all his money?

(1) 20 (2) 24 (3) 30 (4) 32 (5) none of these

Solution:
Let us use the notation to say Vineet can spend Rs 100n in S(100n) ways.

If he buys beer on day 1 then he can spend next Rs 100n-100 in S(100n-100) ways.
If he buys vodka on day 1 then he can spend next Rs 100n-200 in S(100n-200) ways.
If he buys whisky on day 1 then he can spend next Rs 100n-200 in S(100n-200) ways.

=> S(100n) = S(100n-100) + 2.S(100n-200).


=> S(600) = S(500) + 2.S(400), but S(100) = 1 and S(200) = 3.
=> S(300) = S(200) + 2.S(100) = 5, => S(400) = S(300) + 2.S(200) = 11
=> S(500) = S(400) + 2.S(300) = 21 => S(600) = S(500) + 2.S(400) = 43

=> Choice (5) is the right answer

Solution to Quantitative Question # 028


--------------------------------------------------------

The sum of all the divisors of 19^88 - 1 which are of the form (2^a).(3^b) with a, b >
0 is

(1) 168 (2) 224 (3) 360 (4) 744 (5) 1080

Solution:

Here we need to find the max power of 2 and 3 each that divides 19^88 - 1.

Now 19^88 - 1 can be written as (19^11 -1 ).(19^11 + 1).(19^22 + 1).(19^44 + 1).

Since 19 is of the form 3k+1 => each of 19^11 + 1, 19^22 + 1, 19^44 + 1 is of the form 3k+2 and
hence not div by 3. But 19^11 - 1 can be written as (18+1)^11 - 1 = 18^11 + 11C1(18)^10 + ....
11C10(18) => it' div by 9 but not by 27 => max power of 3 is 2.

Similarly, each of 19^11 + 1, 19^22 + 1, 19^44 + 1 is of the form 2k+2 and hence div by 2 but not
by 4. However, 19^11 - 1 = (20-1)^11 - 1 = 20^11 - 11C1(20)^10 + .... + 11C10(20), which means it'
div by 4 but not by 8 => max power of 2 that divides our number is 5.

=> sum of the divisors is (2 + 2^2 + 2^3 + 2^4 + 2^5).(3 + 3^3) = 62.12 = 744.

=> Choice (4) is the right answer

Solution to Quantitative Question # 029


--------------------------------------------------------

Each question is followed by two statements X and Y. Answer each question using
the following instructions:

Choose 1 if the question can be answered by X only


Choose 2 if the question can be answered by Y only
Choose 3 if the question can be answered by either X or Y
Choose 4 if the question can be answered by both X and Y
Choose 5 if the question can be answered by neither X and Y

Let x and y be positive real numbers. Is x^2 + y^2 < 1?


(X) y^3 + y <= x - x^3
(Y) |x| + |y| < √2 and |x| < 1 and |y| < 1

Solution:

y^3 + y <= x - x^3 <= x(1-x^2), buy y + y^3 is positive => 0 < x < 1
Also, y^3 + x^3 + y < x => 0 < y < x < 1.

Now, x^2 + y^2 <= (x-y)/(x+y) + xy, we will do some reverse engineering here
If (x-y)/(x+y) + xy < 1 => x^2 + y^2 < 1. We will prove that (x-y)/(x+y) + xy < 1 or 1 - 2y/(x+y) +
xy < 1 or 2y/(x+y) > xy or 2/(x+y) > x or 2 > xy + x^2 which is true as both x, y < 1. Hence, (X) is
sufficient.

For (Y), drawing graph tells us some values in 1st quadrant remain that doesn't satisfy x^2 + y^2
< 1 e.g. x = 0.8 and y = 0.6, but x = 0.1 and y = 0.1 satisfy => (Y) is not sufficient.

=> Choice (1) is the right answer

Solution to Quantitative Question # 030


--------------------------------------------------------

A cylinder of radius √6 cm and height 3√3 cm is inscribed inside a cube such that
the axis of cylinder is along a diagonal of the cube. The length of side of the cube is

(1) 6 cm (2) 7 cm (3) 8 cm (4) 9 cm (5) none of these

Solution:

The cylinder will touch the face of the cube on its diagonal. Let x be the angle between its diagonal
and the diagonal of the cube along the axis of the cylinder. Then sinx = 1/√3, and let l be the
length of the side of the cube => √3 l = 3√3 + 2√6 cotx => l = 7 cm.

=> Choice (2) is the right answer

Solution to Quantitative Question # 031


--------------------------------------------------------

Consider a rectangular purse of dimension 8 cm X 9 cm. What could be the


maximum radius of two identical coins which can be completely put inside the
purse without overlapping?

(1) 2 cm (2) 2.25 cm (3) 2.5 cm (4) 2.75 cm (5) none of these

Solution:

Let the line joining the centres of two coins be inclined at an angle x to AB => 2R + 2Rcosx = 9,
2R + 2Rsinx = 8 => (9-2r)^2 + (8-2R)^2 = (2R)^2 => R = 5/2.

=> Choice (3) is the right answer

Solution to Quantitative Question # 032


--------------------------------------------------------
If r is the root of f(x) = x^4 + ax^3 - 6x^2 - ax + 1 = 0, then which among the
following is also a root of f(x) = 0?

(1) -1/r (2) (1+r)/(1-r) (3) (r-1)/(r+1) (4) All of these (5) Exactly
two of these

Solution:

If r is the root of f(x) = 0, then f(-1/x) = f(x) tells us that -1/r is also the root of
f(x) = 0 -> (1)

When we are checking that for any r, f(r) = 0 we want to check if f((1+r)/(1-r)) = 0 or not
then (1+x)/(1-x) = y and x = (y-1)/(y+1), on substitution in f(x) we get
f(y) = y^4 + ay^3 - 6y^2 -ay + 1 which is similar to our starting equation -> (2)

The 3rd choice follows from (1) and (2).

Alternative way

put x=-1/x
f(-1/x)=f(x)=x^4 + ax^3 - 6x^2 - ax + 1 = 0

so if r is root, -1/r is also if z is then -1/z is.


clearly also roots taken two at a time is -6
so rz-1-1-r/z-z/r+1/rz=-6
r(z-1/z)-1/r(z-1/z)=-4
(r-1/r)(z-1/z)=-4
now there is no need to go further
z=(1+r)/(1-r) satisfies

=> Choice (4) is the right answer

Solution to Quantitative Question # 033


--------------------------------------------------------

Garibchand decided to sell 89 articles (of same kind). After achieving break even
(on exact no. of articles), he decided to give 10% discount and when he achieved
exact 10% profit, he decided to give 20% discount. Garibchand sold all articles with
20% net profit. Had he not given discount, he would have made profit of

(1)23.6% (2) 25.3% (3) 27.8% (4) 29.2% (5) 31.4%

Solution:

C = CP of an article, S = SP of an article without disc %.


x = units for break even, y = units sold at 10% disc => 89-x-y = units sold at 20% disc.

1 -> 89C = xS
2 -> 89C*1.1 = xS + 0.9S*y
3 -> 89C*(1.2) = xS + 0.9S*y + 0.8S*(89-x-y)

solving 1, 2, 3 we get x = 72, y = 8.

Had there been no discount offered, profit% = [(S-C)/C] * 100 = 1700/72% = 23.61%
=> Choice (1) is the right answer

Solution to Quantitative Question # 034


--------------------------------------------------------

In how many ways can the letters of the word JUPITER be arranged in a row so that
the vowels appear in alphabetic order?

(1) 736 (2) 768 (3) 792 (4) 840 (5) 876

Solution:

Since the order of vowels will always remain the same despite these occupying different positions
-> if we assume each vowel as X then our question is same as asking "arrange JPTRXXX" => in all
7!/3! ways

=> Choice (4) is the right answer

Solution to Quantitative Question # 035


--------------------------------------------------------

The number of ordered (x, y) such that 1/√x + 1/√y = 1/√20 is

(1) 1 (2) 3 (3) 5 (4) 7 (5) none of these

Solution:

The given equation transforms to 1/y = (x+20-4√5.√x)/20x => √5.√x is rational.

=> 5x = (5a)^2, similarly 5y = (5b)^2

=> 1/a + 1/b = 1/2 => (a-2)(b-2) = 4 => (a, b) = (3, 6), (4, 4), (6, 3)

=> Choice (2) is the right answer

Solution to Quantitative Question # 036


--------------------------------------------------------

In a triangle ABC, AB = AC, <A = 80˚ and S is the circumcentre. Bisectors of angles
ACS and ABS meet BS and CS respectively at X and Y. Then < AXY equals

(1) 20˚ (2) 40˚ (3) 50˚ (4) 60˚ (5) 100 ˚

Solution:

Let BY and CX meet at P, at point on AS => SA = SB => <ABS = 40˚ => <ABP = 20˚ => <APB =
120˚, similarly <APC = 120˚ => <BPC = 120˚ => triangle ABP is congruent to triangle XBP => PA
= PX, similarly PA = PY => <APX = <XPY = <YPA => PA = PX = PY => AXY is equilateral.

Hence, <AXY = 60˚

=> Choice (4) is the right answer


The question that was 37th Last Year
---------------------------------------------

Consider a scalene triangle PQR.Points S, T and U are selected on sides QR, PR, and
PQ respectively. The lines PS, QT, and RU meet at point Z. If area(PUZ) = 126,
area(UQZ) = 63, and area(RTZ) = 24, the area of triangle PQR is ?

(a) 324 (b) 351 (c) 360 (d) 364 (e) 378

Consider triangle PQT and PQZ and PZT


The area of PQZ = 1/2 QZ *h = 189
the area of PZT= 1/2 ZT *h =x
so area of PZQ /area of PZT = 189/x = QZ/ZT ----------(1)
Then take triangle QTR,
similarly area of QZR / area of RTZ = y /24 = QZ/QT ------(2)
from 1 and 2---- 189/x = QZ/ZT = y /24

Then take triangle PUR and URQ and apply the same procedure
so we get the rel 24 +x /126 = y /63

Solve and get the value of x and y..


So get total as 351 => choice (b) is the right answer.

Solution to Quantitative Question # 037


--------------------------------------------------------

Pavan had 6 friends in a B-school. At a certain restaurant, he met each of them 12


times, every 2 of them 6 times, every 3 of them 4 times, every 4 of them 3 times,
every 5 twice and all 6 only once. Pavan had dined out alone 8 times without
meeting any of them. How many times had he dined out altogether?

(1) 36 (2) 22 (3) 32 (4) 26 (5) none of these

Solution:

Let set A, B, C, D, E, F are the dining instances when Pavan has dined with his six friends
respectively.

n(A u B u C u D u F) = 6c1 * (no. of times he met each) - 6c2 * (no of times he met two) + 6c3 *
(no. of times he met three) - .... - 6c6 * (no. of times he met all six)
= 6 * 12 - 15 * 6 + 20 * 4 - 15 *3 +6 *2 - 1 * 1 = 28

Therefore total number of times is 28 + 8 = 36

=> Choice (1) is the right answer

Solution to Quantitative Question # 038


--------------------------------------------------------

Let |2x-1| - 3|x+1| = a has two real solutions p and q satisfying 2 <= |p-q| <= 10, then
Max(a) - Min(a) equals
(a) 19/3 (b) 15/2 (c) 35/6 (d) 11/2 (e) none of these

Solution:

Let f(x) = |2x-1| - 3|x+1|


We have f(x) = x+4 if x <= -1,
-5x-2 if -1 < x < 1/2,
-x-4 if x >= 1/2.

Draw the graph of f(x) now and see that the nodes are (-1, 3) and (1/2, -9/2). The shape of this
graph is almost inverted V with V-vertex at (-1, 3) and sligh right shift from (1/2, -9/2).

The equation f(x) = a has no solution if a > 3 because for the line parallel to x-axis doesn't
intersect the graph for y > 3.

The equation f(x) = a has 2 solutions when -9/2 <= a <= 3 and the solutions are determined by
the intersection of line y = a and the lines y = x+4 and y = -5x-2. The solutions are (a-4, a) and (-
(a+2)/5, a).

We are given that solutions p and q are such that 2 <= |p-q| <= 10 => 2 <= |a-4 + (a+2)/5| <= 10
subject to constraint on a => -9/2 <= a <= 4/3
Similarly, the equation f(x) = a has 2 solutions when a < -9/2, and the solutions are determined
by the intersection of line y = a and the lines y = x+4 and y = -x-4. The intersection points would
be (a-4, a) and (-a-4, a).
We are given that solutions p and q are such that 2 <= |p-q| <= 10 => -5 <= a <= -9/2.

Combining the above we get -5 <= a <= 4/3.

=> Choice (1) is the right answer

Solution to Quantitative Question # 039


--------------------------------------------------------

Let f(k) be defined on integer k as f(k) = [k](3) + [2k](5) + [3k](7) - 6k,


where [k](2n+1) denotes the multiple of (2n+1) closest to k. How many values can
f(k) assume?

(1) 4 (2) 7 (3) 10 (4) 13 (5) none of these

Solution:

[k](n) = k + (n-1)/2 - (k+ (n-1)/2) mod n => f(k) = 6 - (k+1) mod 3 - (2k+2) mod 5 - (3k+3) mod 7

=> k +1 = a mod 3, where 0 <= a < 2: 2k +2 = b mod 5, where 0 <= b < 5: 3k +3 = c mod 7, 0 <= c
< 7 => -6 <= f(k) <= 6

=> k = a-1 mod 3, k = -2b -1 mod 5, k = -2c -1 mod 7: By chinese remainder thoerem this has a
solution for any choice of a, b, c => f(k) takes all values in [-6, 6].

=> Choice (4) is the right answer


Solution to Quantitative Question # 040
--------------------------------------------------------

Two cars A and B started from P and Q respectively towards each other at the same
time. Car A was travelling at a speed of 54km/h but due to some problem reduced
its speed by 1/3rd after travelling for 60 minutes. Car B was travelling at a speed of
36km/h. Had the technical problem in car A had arisen 30 minutes later, they would
have met at a distance which is (1/30*PQ) more than towards Q than where they met
earlier(PQ > 120km). Anothet car C starts from P, 90 minutes after car B started at
Q, and car C travels towards Q with a speed of 36km/h, at what distance from P will
cars B and C meet?

(a) 63 km (b) 48 km (c) 40.5 km (d) none of the foregoing

Solution:

Two cars meet at mid-point of PQ if they were at same speed. If the starting point of A
shifts by l1 towards Q and that of
B by l2 towards Q, where l2 < l1), the meeting point shifts by (l1 - l2)/2 towards Q.
=> meeting point shifts by (27-18)/2 km towards Q => PQ = 30*(4.5) km = 135 km

B covers 3/2*36 = 54 km from Q when C starts.


Cars B and C are (135 - 54)km apart when C starts => PR = 81km. If C and B meet =>
they meet in the mid-way as they have
same speed.

=> Choice (3) is the right answer

Solution to Quantitative Question # 041


--------------------------------------------------------

N people vote for one of 27 candidates. Each candidate's vote % is atleast one less
than his/her number of votes. What is the smallest possible value of N?

(a) 108 (b) 127 (c) 134 (d) 162 (e) none of these

Solution:

If a candidate has just 2 votes, then 2/n <= 1/100, so n >= 200. If a candidate has
3 votes, then n >= 150.

So in a minimal solution each candidate must have at least 4 votes. If all have at
least 5, then n >= 135.

If a candidate has 4, then 4/n <= 3/100, so n >= 134. This can be achieved: 1
candidate has 4 votes, the other 26 have 5 each. Then 5/134 = 3.7%, 4/134 = 2.99%.

=> Choice (3) is the right answer

Quantitative Question # 042


--------------------------------------------------------
Each question is followed by two statements X and Y. Answer each question using
the following instructions:

Choose 1 if the question can be answered by X only


Choose 2 if the question can be answered by Y only
Choose 3 if the question can be answered by either X or Y
Choose 4 if the question can be answered by both X and Y
Choose 5 if the question can be answered by neither X and Y

For positive reals x, y, z, Is 1/x + 1/y + 1/z <= 1?

(X) For every quadrilateral with sides a, b, c, d, x.a^2 + y.b^2 + z.c^2 > d^2
(Y) √x + √y +√ z >= √x.√y.√z

Solution:
(X)
Take the 3 sides of the quadrilateral as 1/x, 1/y, 1/z -> since the sum of 3 sides > fourth
side => 4th side can be taken as 1/x + 1/y + 1/z - 1/n where n > 0 and large
We are given that, x/x^2 + y/y^2 + z/z^2 > (1/x + 1/y + 1/z - 1/n)^2 hold true for every n
Take 1/x + 1/y + 1/z = p, and limiting case when n -> Infinity
=> p > (p-1/n)^2 => p < 1.

(Y)
Put 1/x = a, 1/y = b, 1/z = c => √(ab) + √(bc) + √(ca) >= 1
=> 2√(ab) + 2√(bc) + 2√(ca) >= 2
=> By AM-GM on positive numbers, (a+b+c) >= 1 => 1/x + 1/y + 1/z >= 1. Hence, (Y)
is not enough but would have been enough if question asked was, Is 1/x + 1/y + 1/z < 1 or
it was given that at least two of x, y, z are distinct?

Ö Choice (1) is the right answer

Solution to Quantitative Question # 043


--------------------------------------------------------
Consider two cones of heights 1 and 8 units having the same base radii. It is found
that their height is increased by x keeping their vertex angle unchanged, their
volume becomes equal. Then x equals

(1) 2/3 (2) 4/3 (3) 8/3 (4) 16/3 (5) none of these

Solution:
Suppose the radii of two cones to be r initially and r1 and r2 after increasing height by x
since the vertex angle are unchanged
r/1 = r1/(1+x) => r1 = r(1+x)
r/8 = r2/(8+x) => r2 = r(8+x)/8
now the volumes are equal
=> 1/3*pi*r1*r1*h1 = 1/3*pi*r2*r2*h2
=> [r(1+x)]^2 * (1+x) = [r(8+x)/8]^2*(8+x)
=> x = 4/3

Ö Choice (2) is the right answer

44. The question is followed by two statements X and Y. Answer using the following
instructions:

Choose 1 if the question can be answered by X only


Choose 2 if the question can be answered by Y only
Choose 3 if the question can be answered by either X or Y
Choose 4 if the question can be answered by both X and Y
Choose 5 if the question can be answered by neither X and Y

Two vessels A and B having different capacities are partly filled with spirit of
different concentrations. If the content of A is poured into vessel B till it is full, then
the % concentration of B increases by 5%. Is the difference in concentrations of
spirits in containers more than 5%?

(X) initial level of spirit B is greater than that in A

(Y) If the content of B is poured into vessel A till it is full, then the % concentration
of A decreases by 10%

Solution:

Pouring the content of A increases the concentration of B by 5% => A will at least have 5% more
concentration than B. But since B is already partly filled => concentration of A - concentration of
B > 5%. We pick the strongest option as our answer which is already implied in the question and
we don't need to bother checking for (X) and (Y).

=> Choice (5) is the right answer

45. Let F be be mini 4X4 chessboard => it has 16 fields in all. In how many ways is it
possible to select two fields of F such that the midpoint of the segment joining the
centres of the two fields should also be the centre of a field?

(1) 15 (2) 18 (3) 24 (4) 32 (5) none of these

Solution:

The fields of the centre of the fields of the chessboard can be assigned coordinates.
Let the centre of the 1st field in the left most bottom corner be (0, 0) =>
the centres of the fields are of the form (x, y) where 0 <= x, y <= 3.
The coordinate of the midpoint of the segment connecting the points (a,b) and (c,d) are (a+c)/2
and (b+d)/2, and these coincide with the coordinates of the centre of a field if and only if a+c and
b+d are both even.
The point (a,b) may be the centre of any field out of the 16 fields of the mini chessboard. (a,b)
having been selected, the number c can be any of the 2 numbers that have the same parity as a,
and independently of that, d can also have 2 different values. In order to make the chosen points
different, the number of the possible pairs (c,d) is 1 less than 2.2=4. Buthe order of the two
selected points does not matter => the number of the appropriate number pairs is 1/2*(4*4)*3 =
24.

If our question was on NXN square, then it could have been solved in a jiffy with the above logic.
For 6X6 the answer is 144. For 8X8, the answer is 480.

=> Choice (3) is the right answer

46. Let ABCD be a rectangle and E be a point beyond C on AC extended. If < DEB = <
CBE, and AB/BC = 3, then BE/CE equals

(1) √3: √2 (2) √2: 1 (3) √2 + 1: √ 3 (4) 4: √5 + 1 (5) √3+1: 2

Solution:

Treat A (0, 0), B (3, 0), C (3, 1), D (0, 1) and E is (3p, p) as it lies on AC. Let tanI denotes the
Inverse of tan.

Slope of EB = tanI[p/(3p-3)] = 90 - µ (equal angle)


Slope of ED = tanI[(p-1)/(3p)] = 90 -2*µ
=> µ = tanI[(3p-3)/p], and 2*µ = tanI[(3p)/(p-1)].

Eliminiate µ using tan (2*µ) = 2tan(µ)/(1-tan^2(µ))


and we get p as [11 +/- root(11)]/10.

We need to find square root of [(3p-3)^2 + p^2]/[(3p-3)^2 + (p-1)^2] = 2.

You all will be stunned to note that the ratio AB/BC was irrelevant and infact our asked ratio is
always √2:1. Check this in case of a square.

Alternative Solution

Let F be the intersection of the lines BC and ED. The triangle BEF is isosceles, according to the
given information. Let R be the point that is on the same side of line EB as F, and for which the
triangle EBR is isosceles and right-angled, that is, BR=ER and < ERB=90 degrees.

Let, furthermore, T be the intersection of the circumscribed circle of rectangle ABCD with the line
ED. BD is a diameter in the circle, therefore the segment BT is perpendicular to the line ED.
As ACTD is a cyclic quadrilateral, < ETC= < EAD = < ECF. The angles of the triangles ECF and
ETC are pairwise equal, thus the two triangles are similar, and EC:EF=ET:EC, that is,
EC^2=EF.ET.

As < ERB = < ETB = < 90 degrees, the quadrilateral ERTB is also cyclic, and < RTE =< RBE =<
ERF. The triangles ERF and ETR are also similar, thus with the above reasoning we have
ER^2=EF.ET.
We have obtained that EC^2=ER^2=EF.ET, that is, EC=ER, and hence our answer.

=> Choice (2) is the right answer


47. The set S has 5 elements. In how many ways can one select two (possibly
identical) subsets of S whose union is S?

(a) 32 (b) 63 (c) 64 (d) 93 (e) 122

Let the subsets of S be A and B. For each element in S we have three choices (it can belong to
either of A, B or both). That gives each pair of subsets twice except for the case A = B = S. Hence,
we can select 2 subsets in (3^5 + 1)/2 ways.
=> choice (e) is the right answer.

48. The number of real solutions to |1 - |x|| - (1.01)^(1.01x) = 0 is

(a) 1 (b) 2 (c) 3 (d) 0 (e) none of these

At x = 0 we have a solution.
Let f(x) = |1 - |x|| - (1.01)^(1.01x)
f(1) < 0, f(3) > 0 => we have odd number of solutions between (1, 3)
but f(x) is increasing in (1, 3) => we have just 1 solution in (1, 3)
f(-1) < 0 and f(-2) > 0 and f(x) is decreasing in (-2, -1) => 1 solution in this interval also.
Also, f(1000) < 0 and f(3) > 0 and f(x) in (3, 1000) is x - 1 - (1.01)^(1.01x) which is a decreasing
function in this interval => we have one more root in (3, 1000) => 4 roots in all

Ö choice (e) is the right answer.

49. Two budding mathematicians, Srikar and arbit_rageur, play a game. The
computer selects some secret positive integer N < 60 (both Srikar and arbit_rageur
know that , but that they don't know what the value of N is). The computer tells
Srikar the unit digit of N, and it tells arbit_rageur the number of divisors of N.
Then, Srikar and arbit_rageur have the following dialogue:

Srikar: I don't know what N is, and I'm sure that you don't know either. However, I
know that N is divisible by at least two different primes.

arbit_rageur: Oh, then I know what the value of N is.

Srikar: Now I also know what N is.

Assuming that both Srikar and arbit_rageur speak truthfully and to the best of their
knowledge, how many possible values of N are there?

(a)0 (b) 1 (c) 2 (d) 3 (e) none of these

Solution:

Srikar's first statement implies that has a last digit of 0, and must be divisible by 10. Since 10, 20,
30, 40, and 50 have 4, 6, 8, 8, and 6 factors, respectively, arbit_rageur can only be sure of the
value of if he knows has 4 factors. Therefore, the only possible value of is 10.

Ö Choice (2) is the right answer


50. Each question is followed by two statements X and Y. Answer each question
using the following instructions:

Choose 1 if the question can be answered by X only


Choose 2 if the question can be answered by Y only
Choose 3 if the question can be answered by either X or Y
Choose 4 if the question can be answered by both X and Y
Choose 5 if the question can not be answered by combining X and Y also

If 1 < a < 2 and k is an integer, then what is [ak/(2 - a)], where [x] denotes the
greatest integer not larger than x.

(X) [a[k/(2 - a)] + a/2] = p


(Y) [a[k/(2 - a)] + (a+1)/2] = q and k is even

Solution:

Put 2-a = m => 0 < m < 1, we will show that [(2-m)k/m] = [(2-m)[k/m] + (2-m)/2]

[(2-m)k/m] = [2k/m] - k, since k is an integer.

Let us take this case by case.

Case 1: when k/m is an integer = i


[(2-m)k/m] = 2i - k, and [(2-m)[k/m] + (2-m)/2] = [2i -mi + 1-m/2] = [2i -k + 1-m/2] = 2i - k.

Case 2: when i-1/2 < k/m < i, where i is an integer


[(2-m)k/m] = 2i - 1 - k, [(2-m)[k/m] + (2-m)/2] = [2i - m/2 - k] = 2i - 1 - k

Case 3: when i < k/m < i + 1/2, where i is an integer


Prove yourself here that [(2-m)k/m] = [(2-m)[k/m] + (2-m)/2]

=> X is sufficient for all k

Now Y adds 1/2 in X and depending on the value of a[k/(2-a)], we can have multiple (two)
possible value of [ak/(2 - a)] for q.

=> Choice (1) is the right answer

51. Given p and q be positive such that 2 >= p-q, the min value of 2/(p+q) + q/2 is

(1) √2 - 1/2 (2) (√2 + 1)/2 (3) 1 (4) 1/√2 (5) none of these

Solution:

p+k = 2+q where k >= 0


Thus, 2/(p+q) + q/2 = 2/(2+2q-k) + (2q+2-k)/4 + (k-2)/4.
By AM-GM on positive integers, q can be generated based on k
and our expression is >= sqrt(2) + (k-2)/4 which assumes min at k = 0

=> Choice (1) is the right answer


52. Nbangalorekar bought a 10 kg of water-melon in Bangalore that had 99% water.
After the water-melon was left outdoors for a day, it was 95% water. What was the
weight of the dehydrated water-melon?

(1) 9.6 kg (2) 9.5 kg (3) 2 kg (4) can not be determined (5) none of
these
Solution:

Initially, 0.1 kg of dry part + 9.9 kg of water is there. After dehydration, 0.1 kg of dry part id 5% of
total water-melon's weight => weight of dehydrated water-melon is 2 kg.

=> Choice (3) is the right answer

53. Given a trapezium ABCD with AB || CD , CD = 2AB and DB perpendicular to BC.


Let E be the intersection of lines DA and CB, and F be the midpoint of DC. Which
among the following is not true?

(1) ABFD is a rhombus


(2) Triangle CDE is isosceles
(3) If AF and BD meet at G, and GE and AB meet at H then the line DH bisects
segment EB
(4) At least two of the above
(5) none of the above

Solution:

For (1) & (2)


AB/CD = EB/EC = EA/AD => EB= BC and EA = AD
Now, looking at CDE, we see that the BD is a median (EB = BC), and is also perpedicular to EC
(DB at right angles to EC) => the median is also an altitude.
=> the triangle is isosceles with DE = DC (this proves (2))

Now, since A is the midpoint of DE and F of DC, AF||EC


=> DB perpendicular to AF.
Also, we know that AB = DF and AB||DF
=>ABDF is a parallelogram with diagonals intersecting at right angle
=> it is a rhombus.

For (3)
We know that G is the midpoint of AF and BD.
Consider the triangle EDB.
EG is the median to BD, while BA is the median to ED. They intersect at the point H.
=> the third median must also pass through this point.
Joining the third vertex D to H and extending, DH is the median and hence bisects EB.
=> (3) is true.

=> Choice (5) is the right answer

54. 5 professors decide to hold daily meetings such that (i) at least one professor
attend each day (ii) a different set of professors must attend on different days. (iii)
on day N for each 1 <= d < N, at least one professor must attend who was present on
day d. How many maximum days can meetings be held?

(1) 14 (2) 16 (3) 20 (4) 24 (5) none of these


Solution:

We need to find the largest possible number of subsets of {1, 2, 3, 4, 5} such that no 2 subsets are
disjoint. Fix one element from the set to be presented in each subset and we can have 2^4 such
possibilities.

=> Choice (2) is the right answer

55. Square ABCD has side length 6. Circle Q is tangent to sides AB and BC, and is
externally tangent to circle P. Circle P is tangent to sides CD and DA, and is
externally tangent to circles O1 and O2. Circle O1 is tangent to side CD, circle O2 is
tangent to side DA, and circles O1 and O2 are externally tangent to each other and to
circle P. If the radius of circle P is twice the radius of circle Q, and if circles O1 and
O2 both have radius r, then r is (upto 2 places of decimal)

(1) 0.29 (2) 0.36 (3) 0.47 (4) 0.54 (5) none of these

Solution:

Please refer the discussions thread for the solution with figure (post # 227).

Ö Choice (1) is the right answer

56. Given a dart board divided in two regions, one red, one green. If you hit the red
region you get 5 points, if you hit the green region you get y > 2 points. If gcd(5, y) =
1 and let R be the the maximum number of points you can not get for a given choice
of y, but can get R+1 points for same choice of y, then R can not be a

(1) prime (2) composite (3) perfect square (4) two of the foregoing
(5) none of the foregoing

Solution:

By Chicken McNugget Theorem, if gcd(m, n) = 1, then the max R such that am + bn has no
solution in non-negative integers is (m-1)(n-1) - 1. => R = 4y -5 for our choice of values here.

But 4y - 5 = 4k+3 for some k, and perfect squares are of the form 4k or 4k+1.

=> Choice (3) is the right answer

57. If the base 8 representation of a perfect square is ab3c, where a is non-zero, then
c equals

(1) 0 (2) 1 (3) 3 (4) 4 (5) can not be uniquely determinable

Solution:

The perfect square in base 10 is 512a + 64b + 24 + c. The perfect squares are of the form 4k, 8k+1
=> c can be 0, 1 or 4. If c = 0 then 512a + 64b + 24 + c = 4(4t+2) for some t, which can't be a
perfect square. When c = 4, 512a + 64b + 24 + c = 4(4r+3) for some r, which can't be a perfect
square.
=> Choice (2) is the right answer

58. Let 0 ≤ m ≤ n ≤ k ≤ 9 be three integers such that mn + nk + km = 60. The least


possible value of m is

(1) 1 (2) 2 (3) 3 (4) 4 (5) none of these

Solution:

mn + nk + km + m^2 = 60 + m^2 = (m+k)(m+n).

m = 1 gives, 61 = (1+k)(1+n) -> not possible; when m = 2, 64 = (2+k)(2+n) => k = n = 6

=> Choice (2) is the right answer

59. If p, q, r be positive numbers satisfying p + 1/q = 4, q + 1/r = 1, r + 1/p = 7/3, then


pqr =

(1) 2/3 (2) 1 (3) 4/3 (4) 2 (5) 7/3

Solution:

Multiply the 3 equations, and we get pqr + p+1/q + q+1/r + r+1/p + 1/pqr = 28/3 => pqr + 1/pqr
= 2 => pqr = 1.

=> Choice (2) is the right answer

60. The numbers +1 and -1 are positioned at the vertices of a regular 12-gon so that
all but one of the vertices are occupied by +1. It is permitted to change the sign of
the numbers in any k successive vertices of the 12-gon. It is possible to shift the only
-1 to the adjacent vertex if k =

(1) 3 (2) 4 (3) 6 (4) at least two of the foregoing (5) none

Solution:

The key word was "any successive k" -> Let us denote our vertices as
A1, A2, A3, ..., A12 and let -1 be at A1. Let the 4 successive vertices be A1, A4, A7, A10.
The product of the numbers on these vertices is -1 to start with, and if we are changing the sign of
each of A1, A4, A7, A10 then also the product should remain -1. But when we say that we want to
shift -1 to an adjacent vertex, then product on nodes A1, A4, A7, A10 will be 1, which is not
possible.

Similarly, we can argue for k = 6 -> Take successive vertices as A1, A3, A5, A7, A9, A11.

For k = 3, we need even number of touches in 10 nodes and odd number of touches in 2 nodes to
make our case happen. But, this means in all we require even number of touches, or in other
words we should perform our operation of any 3 successive changes even number of times which
in effect means we are back at k = 6, which we proved is not possible.

=> Choice (5) is the right answer


61. Bus A leaves the terminus every 20 minutes, it travels a distance 1 km to a
circular road of length 10 km and goes clockwise around the road, and then back
along the same road to to the terminus (a total distance of 12 km). The journey takes
20 minutes and the bus travels at constant speed. Having reached the terminus it
immediately repeats the journey. Bus B does the same except that it leaves the
terminus 10 minutes after Bus A and travels the opposite way round the circular
road. The time taken to pick up or set down passengers is negligible. A man wants to
catch a bus a distance 0 < x < 12 km from the terminus (along the route of Bus A).
Let f(x) the maximum time his journey can take. The value of x for which f(x) is a
maximum is

(1) 3 (2) 5 (3) 8 (4) 10 (5) none

Solution:

The bus takes 20 mins for 12 km or 5/3 min/km. For x <= 1, the best strategy is to wait up to 10
minutes for a returning bus, so f(x) = 10 + 5x/3. Similarly for 11 <= x <= 12, f(x) = f(12-x).
For 1 < x <= 6, the worst case is that he just misses the right bus, so that the wrong bus comes 10
minutes later and the right bus 10 minutes after that. So he can wait 10 minutes and then travel
12-x km for a total time of 10 + 5(12 - x)/3 = 30 - 5x/3 or he can wait 20 minutes and then travel x
km for a total time of 20 + 5x/3. The first is better for x >= 3. So, summarising:

f(x) = 10 + 5x/3 for 0 ≤ x <= 1


20 + 5x/3 for 1 < x <= 3
30 - 5x/3 for 3 < x <= 6
10 + 5x/3 for 6 < x <= 9
40 - 5x/3 for 9 < x < 11
30 - 5x/3 for x ≥ 11

The maximum value of 25 minutes occurs at x = 3 and x = 9.

=> Choice (1) is the right answer

62. For which positive integer values of n the set {1, 2, 3, ..., n} can be split into n
disjoint elements subsets {a, b, c, d} such that a = (b+c+d)/3?

(1) 6 (2) 12 (3) 16 (4) 36 (5) at least two of the foregoing

Solution:

Since we have 4 distinct elements in each subset, 'n' shud be divisible by 4.


directly option (1) is eliminated.
now let a1, a2, a3.... be the different values 'a' can take.
so we get 4*(a1+a2+a3......)=n(n+1)/2 {since all subsets r disjoint}
so n(n+1) shud be divisible by 8
answer can be only option (3)

=> Choice (3) is the right answer

63. Let ABC be a triangle and D and E be internal points on BC and AC respectively.
BD/DC = EA/CE = 1/2. O is the intersection of AD and BE. If the area of triangle ABC
is 2 sq. unit, then area of quadrilateral ODCE (in sq. unit) is
(1) 4/5 (2) 14/15 (3) 16/15 (4) 6/5 (5) none of these

Solution:

Area(ODCE) = ar(DCE) + ar(ODE) = 8/9 + ar(ODE)

ar(ODE) = 0.5*DE*OM (OM is the altitude of ODE)

OM = 2*(altitude of ABDE)/5= 2*(1/3 * altitude of ABC) /5 = 2(altitude of ABC)/15


ar(ODE) = 8/45

ar(ODCE) = 16/15

=> Choice (3) is the right answer

64. Katrina and Deepika have some marbles with each of them, such that the
number of marbles with Deepika is thrice that with Katrina. If Katrina distributes
her marbles equally among certain number of bags, then she is left with 31 extra
marbles. If Katrina and Deepika were to pool the marbles and then distribute the
total marbles equally among the same number of bags as Katrina did, they will be
left with 16 marbles. The number of marbles with Deepika is the largest possible
three digit number. How many bags are needed to equally divide all the marbles
with Deepika, if the number of those bags is the smallest possible two digit number?

(1) 11 (2) 13 (3) 29 (4) 31 (5) none of these

Solution:

Let 3x and x be the number of marbles with deepika and katrina.


Now using the conditions given in the problem the following two equations can be formed..
x=nk+31 (Implies n>31)
4x=np+16 (n is the number of bags)

Now n(p-4k)=108 and n >31..so=> n = 36, 54 or 108 as n has to be div by 108 and n > 31.

=> Number of marbles with Deepika is 3*(36*8 + 31) = 957 for n = 36; for n = 54, 108 we have
lesser value.

957 = 3*11*29

=> Choice (1) is the right answer

65. If all palindromes (positive integers which is unchanged if you reverse the order
of its digits) are written in increasing order, what is the possible number of prime
values can the difference between successive palindromes take?

(1) 0 (2) 1 (3) 2 (4) 3 (5) none of these

Solution:
Let x be a palindrome and x' the next highest palindrome. If x < 101, then it is easy to see by
inspection that x' - x =
1, 2 or 11, so the only prime differences are 2 and 11.

So assume x > 100. If x and x' have the same final digit, then their difference is divisible by 10 and
hence not prime.
So they must have different digits. Thus either x = d9...9d and x' = d'0...0d', where d < 9 and d' =
d+1, or x' has one more digit than x and d = 9, d' = 1. In the first case x' - x = 11. In the second case
x' - x = 2. So again the only prime differences are 2 and 11.

=> Choice (3) is the right answer

66. Which among the following is true? (Mark the strongest option)

(1) Given 8 natural numbers, none greater than 15 => at least 3 pairs of them will
have same positive difference
(2) Among the numbers x, 2x, . . . , 5x, there is one that differs from an integer by at
most 1/6
(3) A warehouse contains 200 boots of size 41, 200 boots of size 42, and 200 boots
of size 43. Of these 600 boots, 300 each are left and right boots => Implex can find
among these boots at least 100 usable pairs
(4) All of the above
(5) Exactly two of the above

Solution:

We have 8C2 = 28 pairs of differences -> the values it can take is from 1 to 14. But 14 can't be
more than once => at least 27 pairs of differences can be from 1 to 13 => 1 difference takes at least
3 values => (1) is true

(2) is left for student's to prove.

There are two categories into which we can fit the three sizes; those sizes
which are more right boots than left boots, left = right, left > right.
=> the two sizes lie in the same catgory. Let us say that sizes 41 and 42 have more right boots than
left boots (an analogous argument will hold if two sizes have more left boots than right boots).

We have 300 boots in all, and at most 200 left boots in any one size.
=> sum of the left boots in any two sizes is at least 100. We have shown that there are at least 100
left boots in sizes 41 and 42, and each of these sizes contains more right boots than left boots.
Hence, (3) is true.

=> Choice (4) is the right answer

67. Let -2 < x < 3, 0 < y < 4, 2 < z < 5. If (3-x)(4-y)(5-z)(3x+4y+5z) achieves the
maximum possible value then which among the following is not true?

(a) 3x+4y = 0 (b) |x| < |y| (c) z = 5/2 (d) two of the foregoing (e) none

Solution:

(3-x)(4-y)(5-z)(3x+4y+5z) = 1/60*(9-3x)(16-4y)(25-5z)(3x+4y+5z). Take 9-3x = A, 16-4y = B, 25-


5z = C, 3x+4y+5z = D. A+B+C+D = 50 => ABCD is max when A=B=C=D. Solving we get x = -7/6,
y = 7/8, z = 5/2.
=> Choice (2) is the right answer

68. If a, b, c, d, e, f are six positive real numbers such that a, b, c are in AP, d, e, f are
in HP, and ad, be, cf are in GP (with common ratio not equal to 1) then a : b : c is

(a) 1/d : 1/e : 1/f (b) f : e : d (c) d^2 : e^2 : f^2 (d) (e+f) : (d+f) : (d+e) (e)
none

Solution:

Use e = 2df/(d+f) and b = (a+c)/2


Put this is (ad)(cf) = (be)(be)
=> (a+c)^2/ac = (d+f)^2/df => a/c + c/a = d/f + f/a, thus a/c can be d/f or f/d, but for a/c = f/d
we get common ratio of GP as 1.
=> a/c = d/f (you can reach this using componendo and dividendo on the above)

Now, b = (a+c)/2 => a:b:c


= d : (d+f)/2 : f = d : df/e : f = 1/f : 1/e : 1/d

=> Choice (5) is the right answer

69. In a chess tournament each player plays every other player once. A
player gets 1 point for a win, ½ point for a draw and 0 for a loss. Both men and
women played in the tournament and each player scored the same total of points
against women as against men. The total number of players in the tournament can
be

(a) 18 (b) 25 (c) 32 (d) 42 (e) 45

Solution:

Let x be the number of men and y be the number of women. Total number of matches being
played in the tournament are (x+y)C2. Men play xC2 among themselves, women play yC2
amongst themselves, and men play xy against women.
=> xC2 + xy + yC2 = (x+y)C2.
But since, each contestant scores same number of points against men as against women => xC2 +
yC2 = xy. Thus, 2xy = (x+y)C2 => 4xy = (x+y)*(x+y-1) => (x-y)^2 = x+y => Total number of
contestants is a perfect square.

=> Choice (b) is the right answer

70. A circle C1 of radius x touches other two circles C2 and C3 of radii y and z (both <
x), the centres of 3 circles being on the line (C1 being in the middle). If the common
tangents of C1, C2 and C1, C3 are perpendicular, then (1+ √(y/x))(1+√(z/x)) =

(a) √2 (b) √3 (c) 2 (d) can not be determined (e) none

Solution:

Let us draw the figure for this. Let common tangent to C1 and C2 be segement PQR where, P lies
on C2, Q on C1. Let common tangent to C1 and C3 be STR where S lies
on C3, T on C1 and R is the intersection point of perpendicular tangents.
Let the centres of circles C1, C2, C3 be M, N, P respectively.
The parallel to PQ from N meets MQ at T.

Look at this -> cos A = (x-y)/(x+y) [in triangle TMN], where A = < QMN, and sin A = (x-z)/(x+z)
[as <TMP = 90 - A].

=> (x-y)^2/(x+y)^2 + (x-z)^2/(x+z)^2 = 1.


=> (x-y)/(x+y) = 2√(yz)/(y+z)
=> √(y/x) = (√x - √z)/(√x + √z)
=> (1+ √(y/x))(1+√(z/x)) = 2

=> Choice (c) is the right answer

71. For a balanced diet Khiladi Akshay Kumar needs to take pulse between 1/2 kg
and 1 kg for every 3 kg of rice. Let there be an unequal percent rise in the prices of
rice and pulse. Assuming that Akshay doesn't take unbalanced diet and keeps his
total amount of consumption of rice and pulse same as earlier, the maximum
percent rise in the consumption of rice Akshay can make will be about

(a) 8% (b) 14% (c) 22% (d) can not be determined (e) none

Solution:

Since maximum rise is there


we need to assume that initially Akshay took 1 kg of pulse for every 3 kg of rice
and after price rise he takes 1/2 kg of pulse for every 3 kg of rice

now initially he eats x kg rice and x/3 kg pulse


Let akshay consumes y kg of rice after price rise so total consumption = y+y/6
x+x/3=y+y/6
y=8x/7
this will lead to 14% rise if the prices rise by equal percentage.

=> Choice (b) is the right answer

72. Let 2 numbers be given in each OPTION below

(a) A = 31^11, B = 17^14


(b) C = 2^100 + 3^100, D = 4^80
(c) E = (1-1/4)(1-1/9)(1-1/16)….(1-1/100^2) , F = 1/2
(d) G = 1/2 - 1/3 + 1/4 - 1/5 + .... + 1/100, H = 1/5

Which among the set constitutes the greater number of the two in the pair in (a) to
(d)?

(1) (B, D, F, H) (2) (B, C, E, G) (3) (A, D, F, H) (4) (B, D, E, G) (5) none of these

Solution:

B > A. How? 31^11 < 32^11 = 2^55 < 2^56 = 16^14 < 17^14

D > C. How? 4^80 has [80log4 +1] = 49 digits. 3^100 has [100log3 + 1] = 48 digits. 2^100 has 31
digits.

Alternate method:
We will show that 4^80 > 2.(3^100). Remember, (1+x)^n >= 1+nx for x >= -1 and n >= 1. Thus,
we need to show if (256/243)^20 > 2 -> look at this, 256/243 = 1+1/20 => (256/243)^20 >
(1+1/20)^20 >= 2. We are dome here!

E > F. How? E = 1.3.2.4.3.5.4.6.5.7.6.8.7.9.8.10.9.11/(2.3.4.5.6.7.8.9..10)^2


E = 11/20 = 0.55 and F = 0.50

G > H is simple. How? Club the two terms each i.e. (1/2-1/3) + (1/4-1/5) + ... 1/100 = 1/6 + 1/20
... > 13/60 > 1/5.

=> Choice (4) is the right answer

73. In a certain class of 300 students , the number of students who either do not
study at home or do not attend classes is a third more than of those who either study
at home or attend classes. the number of students who do not study at home but
attend classes is two fifths more than those who study at home but do not attend
classes, while the number of students who study at home as well as attend classes is
half of those who neither study at home nor attend classes. If the number of
students who only study at home or only attend classes is a third less than those
who do either, then how many students who either do neither or do both?

(1) 120 (2) 150 (3) 180 (4) 210 (5) none of these

Solution:

Let a be the number of students who do not study at home and do not attend classes.
Let b be the number of students who do not study at home but attend classes.
Let c be the number of students who study at home and attend classes also.
Let d be the number of students who study at home but do not attend classes.
Given d+b = 2/3(b+c+d) and d+c+b = 3/4(a+b+d)
if d+b = 2x => b+c+d = 3x, and a+b+d = 4x
Also, (a+b+d) + (c+b+d) - (b+d) = 300
=> x = 60

=> a+c = 180

=> Choice (3) is the right answer

74. Yuvraj, Rohit and Mahender each had age (always considered an integer) less
than 100, such that sum of the ages of any two of them is same as reverse of third's
age. Which among the following can not be true?

(1) The sum of the ages of the three is always 99.


(2) Yuvraj's age, strictly middle in age among the three, can assume 8 values
(3) If Rohit was older than either of the others, the youngest he could be is 45
(4) At least two of the above
(5) none of the above

Solution:

Let the ages of Yuvraj, Rohit and Mahender be : A, B and C respectively.


Given : A, B, C <= 99
And some of any two is reverse of the 3rd => sum of two is 9's complement of the third => A+B =
99 - C
=> A+B+C = 99.
Hence, (1) is true.

Hence, the first and second digits of the sum of two have to be 9's complement of each other.
Thus, we have only 10 possibilities :
(1st Digit of A+B, 2nd Digit of A+B) = (SUM) ==> (C) - (Yuvraj,B)
(0,9-0) = (09) ==> (90) --> Not a solution, since we can't split 9 further (only poss. is 9 and 0, but
that is trivial, hence not valid)
(1,9-1) = (18 ) ==> (81) - (9,9) Hence, Yuvraj cannot be 9
(2,9-2) = (27) ==> (72) - (18,9)
(3,9-3) = (36) ==> (63) - (27,9) {(18,18 ) isn't valid for Yuvraj}
(4,9-4) = (45) ==> (54) - (27,18 ),(36,9)
(5,9-5) = (54) ==> (45) - (36,18 ){(27,27) again isn't valid)
(6,9-6) = (63) ==> (36) - In this case, Yuvraj is 36
(7,9-2) = (72) ==> (27) - Again Yuvraj is 27
(8,9-1) = (81) ==> (18 )- (27,54), And Yuvraj can be 18
(9,9-9) = (90) ==> (09) --> Again not a valid solution...

But from the above deductions, for him to be strictly aged in between, he cannot take 8 different
ages,
=> (2) is false,

(3) is true is easy to check.

=> Choice (2) is the right answer

75. N < 100 Miss Universe contestants of 2010 are standing in a circle and numbered
from 1 to N. Starting counting from 1 initially, in succession, every second one is
removed from the contest and eliminated, and the last one is declared the winner. If
contestant number 25 wins eventually, then how many values can N take?

(1) 3 (2) 1 (3) 2 (4) 0 (5) 4

Solution:

Let the number of contestants be 2^n where n is non-negative => for n = 0, 1 the winner is 1. For
n =2, 3 winner is 1. It is easy to prove that for any n here (2^n) contestants, the contestant
number 1 is the winner always.

Let the number of contestants be 2^n + M where 0 < M < 2^n. Thus, after first M eliminations
i.e. passing through 2M contestants and being at 2M+1 (which will be spared for elimination), we
have 2^n contestants and (2M+1)th is at 1st position, and will be the winner. Thus, the winner for
k number of contestants is 2(k-2^m) + 1 where m is largest integer such that k > 2^m.

Thus, for N = 12 + 16 = 28, 12 + 32 = 44, 12 + 64 = 76.

=> Choice (1) is the right answer

76. A rectangular block L x 100 x H, with L ≤ 100 ≤ H, where L and H are integers, is
cut into two non-empty parts by a plane parallel to one of the faces, so that one of
the parts is similar to the original. How many possibilities are there for (L, H)?

(1) 10 (2) 12 (3) 20 (4) 24 (5) none of these


Solution:

We must cut the longest edges, so the similar piece has dimensions L x 100 x k for some 1 ≤ k < H.
The shortest edge of this piece cannot be L, so it must be k. Thus L x 100 x H and k x L x 100 are
similar. Hence H = 100^2/L, k = L^2/100. Now 100 = 2^4•5^4, so 100^2 has 25 factors, of
which (25-1)/2 = 12 are < 100.

=> Choice (2) is the right answer

77. ABCDE is a pentagon such that < A = < B = < D = 120˚, and < C = < E. Let BC = 1
and CD = √3. If a circle can be inscribed in the pentagon, then its radius will be

(1) 4 - 2√3 (2) 3 - √3 (3) 3√3 - 4 (4) 4√3 - 6 (5) none of


these

Solution:

The construction of the pentagon is very much valid here.


Let's see if we can inscribe a circle in this pentagon.

tan (< DBC) = √3/1 => < DBC = 60 degrees.


Let O be the centre of the circle and P and Q be tangency points of CB and AB respectively => By
RHS, triangles OQB and OPB are congruent and hence < OBP = 60 degrees, but since < DBP = 60
degrees, we are effectively saying that DB passes through the centre of the circle.

What you get is R > 1 [3/2(√3-1)], when clearly it should be < 1. Thus, no such construction of the
circle is possible.

=> Choice (5) is the right answer

78. Let [x] denotes the greatest integer less than or equal to x. The number of
positive reals x such that 9x/10 = [x]/( x - [x] ) is a

(1) perfect square (2) prime (3) perfect cube (4) perfect number
(5) none of these

Solution:

Let x = I + f, where I is non-negative integer and 0 < f < 1 [for f = 0, 1 we have no solution]

Thus, what you get is 9(I+f)/10 = I/f. This is a quadratic in f whose only one root will be in (0, 1),
as the product of the roots of 9f^2 + 9If - 10I = 0 is negative, the other root is negative.

The positive root is [-9I + root(81I^2 + 400I)]/18 and is < 1. Solving for this, we get eight
legitimate I from 1 to 8.

=> Choice (3) is the right answer

79. 7 IIMs participate in a B-schools sports meet, where the use of expletives is
officially prohibited. Each team from an IIM plays against the other exactly once.
What is the possible minimum number of matches that could have been played so
far such that among every 3 teams, atleast two have played against each other?
(1) 9 (2) 10 (3) 14 (4) 15 (5) none of these

Solution:

Treat each team as a point (vertex), and denote matches between two teams as the segment
joining these two points. If we isolate these teams into a set of 4 and 3 each, and draw the
complete graphs (a quadrilateral with diagonals also joined, and another disjoint triangle), then
we have 4C2 + 3C2 = 9 edges (matches) in all. Our condition is satisfied. Can we do better than 9?
The answer is NO, and left as an exercise for students to prove.

=> Choice (1) is the right answer

80. In a triangle ABC, AC = 3AB. From C, CD is drawn perpendicular to the bisector


of < A. If AD intersects BC at X, then AX/XD is

(1) 1:1 (2) √2: 1 (3) √2 + 1: 2 (4) √3 - 1: 1 (5) √3: √2

Solution:

Draw the exernal bisector of < A, meeting CB extended at E => < EAX = 90 degrees.

By internal and external angle bisector rules, we have EB = 2y, BX = y, XC = 3y as BX/XC =


EB/EC = AB/AC = 3 => EX = XC.

Thus, we have congruent triangles AEX and DCX. Thus, AX = XD.

Alternate solution:

Let AB=x=>AC=3x, Also, <ADC=90

We also know AX= 2.AC.BC.cos(A/2)/(AC+BC)

AX=2.3x^2 cos(A/2)/4x, AX=3x cos(A/2)/2

From tria ADC, cos(A/2)=AD/AC=AD/3x

Simplifying, 2AX=AD

=> Choice (1) is the right answer

81. Gripened comes back to India after earning good bucks from Korea and heads
for a shop in Bangalore having expensive shirts. The price of each shirt is a multiple
of ten in Rs and is marked in K (in Rs) i.e. multiple of thousand e.g. if the price is Rs
2670, then it's marked as 2.67 K. Gripened buys 4 shirts and goes to the counter for
the bill. The counter-boy multiplies the marked price of 4 shirts and hands gripened
the bill of 7.11 K. Gripened being smart realizes the mistake and asks for the new bill
in which the marked price of each shirt is added. To Gripened's astonishment, the
bill this time also comes as 7.11 K. What was the marked price in K of the least
expensive shirt that Gripened bought?
(1) 1 (2) 1.2 (3) 1.5 (4) can not be determined (5) none of these

Solution:

Let the MP in Tens (Rs) of the four shirts be a, b, c, d. Then a, b, c, d are whole numbers with

a + b + c + d = 711 = 32 × 79 and (a/100).(b/100).(c/100).(d/100) = 711/100.

=> abcd = 711 × 10^6 = 2^6 × 3^2 × 5^6 × 79. Exactly one marked price is a multiple of 79, and
at most three prices (in Tens Rs) are even or are a multiple of 5.

It is not possible for three prices to be a multiple of 25. Otherwise, the remaining price would be
the multiple of 79, and the sum of the three remaining prices would also be a multiple of 79 as
well as of 25.
But 79 × 25 > 711, and this is not possible. Hence, at least one of the prices is a multiple of 5^3 =
125; this price is clearly not a multiple of 79.

Case 1: One of the prices is 5×79 = 395. Suppose that a = 5×79 = 395. Suppose that b is a multiple
of 5^3 = 125. Since, not all four MPs can be a multiple of 5, one price, c, say, must be a multiple of
5^2 = 25. If (a, b) = (395, 125), then, modulo 25, a+b+c = 20. Since d can have only 2, 3, 5 as
prime divisor, d = 16. But this leads to c = 175 = 7×5^2, which is not possible. If (a, b) = (395,
250), again d = 16 so that c = 50 = 2×2×5^2. But then abcd is not divisible by 3. Since a+b < 711,
this exhausts the possibilities and Case 1 cannot occur.

Case 2. One of the prices, say a is one of the multiples 79, 158, 231, 316, 474 of 79 and another, say
b is one of the multiples 125, 250, 375, 500, 625 of 125. Examining the cases and conducting an
analysis similar to that of Case 1, we arrive at the unique solution (a, b, c, d) = (316, 125, 150, 120)
= (2^2 × 79, 5^3, 2 × 3 × 5^2, 2^3 × 3 × 5)

Therefore, the MP of four shirts is 1.2 K, 1.25 K, 1.50 K and 3.16 K.

=> Choice (2) is the right answer

82. Each question is followed by 2 statements X and Y. Answer each question using
the following instructions

Choose A if the question can be answered using X alone


Choose B if the question can be answered using Y alone
Choose C if the question can be answered using either X or (exclusive) Y
Choose D if the question can be answered using X and Y together
Choose E if the question can not be answered using X and Y also

A PaGalian alphabet consists of consonants and vowels. The rule is that a finite
sequence of letters in a word alternates between consonants and vowels. How many
letters are there in the alphabet?

(X) There are 4800 five letter words

(Y) The difference between the number of five and four letter words is between
1500 and 1800

Solution:
Let the alphabet has c consonants and v vowels. Our 5 letter word can have c*v*c*v*c + v*c*v*c*v
words. From X we have 4800 = c^2.v^3 + v^2.c^3 = c^2.v^2.(c+v) = 3.2^6.5^2 => c or v is a
multiple of 5. Let c = 5, => v can be 2^2 = 4, doesn't satisfy; if c = 10, and v = 2, this fits in good,
and we can have no other case here => c+v = 12. (X) is good enough to answer us!

What we have with (Y) is 1500 < F (c, v) = c^2.v^2(c+v-2) < 1800.
Now, F(c, v) is increasing function w.r.t. c and v. Thus, if we take c = v, we have c^4.(2c-2) < 1800
=> c can at most be 4.

Now, take c = 1, 2, 3, 4 each and we are done.


For c = 1, 1500 < v^2.(v-1) < 1800 => v = 12 => alphabets can be 13
For c = 2, 1500 < 4v^3 < 1800, but 4.7^3 < 1500 and 4.8^3 > 1800. No solution.
For c = 3, 1500 < 9v^2.(v+1) < 1800, F(3, 5) < 1500 and F(3, 6) > 1800
For c = 4, v = 4 satisfies => 8 alphabets => (Y) is not sufficient.

=> Choice (A) is the right answer

83. Problems 81, 82 and 83 were posed in QQAD 2008. 25 students solved at least
one of the three. Amongst those who did not solve 81, twice as many solved 82 as 83.
The number solving only 81 was one more than the number solving 81 and at least
one other. The number solving just 81 equalled the number solving just 82 plus the
number solving just 83. How many solved just 83?

(1) 2 (2) 5 (3) 6 (4) can not be determined (5) none of


these

Solution:

Let a solve just 81, b solve just 82, c solve just 83, and d solve 82 and 83 but not 81. Then 25 - a - b
- c - d solve 81 and at least one of 82 or 83. The conditions give:
b + d = 2(c + d); a = 1 + 25 - a - b - c - d; a = b + c.
Eliminating a and d, we get: 4b + c = 8. But d = b - 2c ≥ 0, so b = 6, c = 2.

=> Choice (1) is the right answer

84. Let x, y, z be three distinct integers such that x < y < z and 3x^2 – (y+z)x + 2(y-
z)^2 = 0. The minimum positive value of z will be

(1) 2 (2) 3 (3) 5 (4) 8 (5) none of these

Solution:

This is a completely raw problem where it's better to use your weirdo skills, substitution methods
to get at the desired answer. Please read the solution by students on the thread. The thing to see
quickly here is that x can not be negative, and if x = 0 then y = z. Thus, our 1st starting point is
that z can at least be 3. Put z = 3 in our equation and we have valid x and y.

=> Choice (2) is the right answer

85. Let a, b, c be real such that a+b+c ≠ 0. If


(a^5+b^5+c^5)/((a^2+b^2+c^2)(a^3+b^3+c^3)) = 5/6, then ab(a+b) + bc(c+a) +
ca(c+a) - 3abc equals
(1) 1/2*(a^3 + b^3 + c^3)
(2) (a^2 + b^2 + c^2)^2/(a+b+c)
(3) a^3 + b^3 + c^3
(4) 3*(a^3 + b^3 + c^3)
(5) (a^4 + b^4 + c^4)/(a + b +c)

Solution:

Put a = 1 and b = 0 the equation (a^5+b^5+c^5)/((a^2+b^2+c^2)(a^3+b^3+c^3)) = 5/6


reduces to (c+1)^3*(c^2 - 3c + 1) = 0 => c^2 - 3c + 1 = 0 as a+b+c ≠ 0.
We are being asked to find ab(a+b) + bc(b+c) + ca(c+a) - 3abc = c(1+c) [this is what we need to
find]
Looking at the choices (a) becomes 1/2*(1+c^3), by putting a = 1, b = 0.
Multiplying both sides of c^2 - 3c + 1 = 0 by c + 1 we get c(1+c) = 1/2*(1+c^3)

The idea is that it might not strike you immediately as to how to resolve expressions, and here is
when it helps putting some manageable values, that satisfies conditions of the problem. What we
have got here in terms of c is something that need not be solved. It just needs to be equated with
answer options quickly.

Also, try the same approach by putting a = -b, we will reach to (1) only.

=> Choice (1) is the right answer

86. A manufactured cloth piece comes in a fixed length, say L. The piece can be sold
in at most two parts, however it becomes difficult to sell if the cloth is cut.
A shopkeeper sells it with selling price which is directly proportional to the length x
of cloth piece bought upto L/2 and after that (x >= L/2) he also charges the cost
price of the remaining unsold part. If the selling price of cloth piece bought of
length x >= L/2 is directly proportional to (x+L), then for x <= L/2, the profit % on
the sale of the cloth is

(1) 50% (2) 75% (3) 100% (4) can not be determined (5) none

Solution:

SP = Ax (x <= L/2)
= Bx + P(L-x) (x >= L/2)

At x = L/2, AL/2 = BL/2 + PL/2 => A = B + P --(1)

Using the info given: K(x+L) = Bx + P (L-x) = (B-P)x + P(L)


A case that satisfies would be B = 2P => A = 3P

Hence, for x <= L/2, SP = Ax = 3Px, CP = Px


=> Profit = 200%

=> Choice (5) is the right answer


87. Five students Implex, Slam, Sanyo, dewan and nbangalorekar are wearing caps
of Blue or Green color without knowing the color of his own cap. It is known that
the students wearing the Blue cap always speaks the truth while the ones wearing
Green always tell lies. If the students make the following statements

Implex: I see 3 blue caps and one Green


Slam: I see 4 Green caps
Sanyo: I see 1 Blue cap and 3 Green
dewan: I see 4 Blue caps

Then, which among the following (Student, Cap Color) combination is correct?

(1) (Implex, Blue) (2) (Slam, Green) (3) (dewan, Blue)


(4) at least two of the foregoing (5) none of these

Solution:

Statement - dewan: I see 4 Blue caps


If dewan is wearing a Blue Cap that means, everyone is wearing a Blue Cap. Not Possible. So
dewan is wearing a green cap.

Now come to statement: Implex: I see 3 blue caps and one Green
If implex wears a Blue Cap then he sees i guy with dewan with green and rest with blue. so Slam
wears Blue but he sees 4 green cap. Not possible again.
So Implex also wears a Green Cap.

left cases -
Implex___Slam___Sanyo___dewan___nbangalorekar ____Case
G_______G______B________G________B___________Satis ifes

G_______B______G________G________G_____Rejected(as sanyo is wearing green but


still speaking truth.)

G_______B_______B________G______G___Rejected.

so only statement holds true. (2) (Slam, Green)

=> Choice (2) is the right answer

88. If f(x) = x^2 - 2x then for how many distinct real α is f(f(f(f(α)))) = 3?

(1) 3 (2) 6 (3) 5 (4) 9 (5) none of these

Solution:

f(f(f(f(α))))=3

For f(α)=3, x=3 or -1


Since 3 is repeated now, we just need to check for -1

For f(α)=-1, α=1

For f(α)=1, α=(1+√2) or (1-√2)

For f(α)=(1+√2), α=(1+√(2+√2)) or (1-√(2+√2))


For f(α)=(1-√2), α=(1+√(2-√2)) or (1-√(2-√2))

For each of the above highlighted values of α, the expression will give a result =3

The important aspect to notice here is that f(3)=3 and hence the moment we get answer as 3, the
cycle will not affect it. Eg. α=1, after 2 cycles will give 3 and that will always give the answer as 3.

=> Choice (4) is the right answer

89. In triangle ABC, M is the mid-point of BC. If < AMB = 45˚, and < ACM = 30˚, then
< BAM is
(1) 30˚ (2) 45˚ (3) < 30˚ (4) > 45˚ (5) none of
these

Solution:

Take the point D on AC such that BD is perpendicular to AC. Why? Because in 30-60 right angle,
we know a side is half the hypotenuse and the question gives a big HINT by telling that M is the
mid-point of BC. Thus, there has to be something in us to explore this kind of construction.
Join DM.

In triangle DBC, we already have angles BDC and ACB => < DBM = 60 degrees. By sine rule on
DBC we have BD = BC/2 = BM => triangle BDM is equilateral.
Thus, < DMB = 15 degrees, and it triangle ADM, < AMD = 60 -45 = 15 degrees.
=> AD = BD = DM, D being the common point, we see a hope here! Convert the hope into action
by treating D as centre of the
circle that passes through A, B and M.
Use the well known rule that the angle subtended by the arc of a circle at the centre is twice of
angle subtended at the circumfrence. Thus, 1/2* (< BDM) = 1/2*60 degrees = < BAM.

=> Choice (1) is the right answer

90. Let S be a subset of {1, 2, 3, ... , 15} such that no two subsets of S have the same
sum. What is the largest possible sum for S?

(1) 55 (2) 58 (3) 61 (4) 63 (5) none of these

Solution:

To find S with sum of the elements having max sum, we can start with 15 and 14 safely. 13 can also
be taken
without any harm.
Now, 15 = 15 (what a good identity!), 14 = 15-1, 13 = 15-2.
The next number will be 15-4 as if next were 15-3, then 15-3 + 15 = 15-1 + 15-2 [condition
violated].
Now, we have 4 numbers with us. Each of the next number below 15-4 i.e. 15-5 can't be in our set
as
15-5 + 15 = 15-1 + 15-4, 15-6 can't also be there as 15-6 + 15 = 15-2 + 15-4.
But we have no such problems with 15-7. The set till now is {15, 14, 13, 11, 8}.
Any new element 15-x, where x > 7 can be combined with some of the elements in {15, 14, 13, 11,
8} whose sum will be equal to some of the rest elements. We stop here!

Thus -> 0, 1, 2, 4, 7 became seed (for the namesake) elements which needed to be subtracted from
15.
The next seed element will be 13 as any number between [8, 12] will violate condition.
e.g. 8+0 = 7+1 or 11+0+1 = 7+4+1 -> note that we must have same number of summands on
either side to do this kind of generation.
The 1st few seed elements are 0, 1, 2, 4, 7, 13, 24, 46, 86.

=> Choice (3) is the right answer

91. Two identical marked dices are brought together and kept with one of their faces
in full contact. How many different arrangements are possible?

(1) 36 (2) 60 (3) 72 (4) 84 (5) none of these

Solution:

Keeping one face of DIE1 constant, we can bring it in contact with each of the 6 faces of DIE2 in 4
possible ways. While bringing a face of DIE1 in contact with the same face of DIE2 (same
number), each of the generated arrangements will be unique. But when bringing a face of DIE2
with a different face (different number), each arrangement would be repeated once.

This leads to the Number of possible arrangements = (6x4) + (6x5x4/2)


= 24 + 60

=> Choice (4) is the right answer

92. Let the quadratic ax^2 + bx + c be such that a, b, c are distinct and each of a, b, c
belong to {1, 2, 3, ..., n} such that x+1 divides ax^2 + bx + c.
If the number of such quadratic polynomials are < 99, then max (n) is

(1) 14 (2) 15 (3) 16 (4) 18 (5) none of these

Solution:

For the set 1 to N, the sets (a,b,c) can be formed as:


1 with (2 to N-1) will give 'c' less than or equal to N => N-1-2+1 = 2(N-2) sets
2 with (3 to N-2) => 2(N-4) sets
3 with (4 to N-3) => 2(N-6) sets and so on.
(factor of two added to account for a and c getting interchanged)

We divide into two cases from here:


case N = 2x (even)
=> total number of sets = 2 [2 + 4 + .. + (2x-2)] = 2x(x-1)
=> 2 (x^2 -x) < 99
=> x = 7 (max)
=> N = 14

case N = 2x+1
=> total number of sets = 2 [1+3+..+(2x-1)] = 2 x^2
=> 2.x^2 < 99
=> x = 7 (max)
=> N = 15

=> Choice (2) is the right answer


93. Yana and Urvashi normally work for 8 hours but do extra four hours on
overtime. A job was done by 2 ladies together in 12 days in which Yana and Urvashi
did overtime for few days (not necessarily same number of days). It is calculated
that had Yana not done her overtime, then the work would have taken 15 days to
complete and corresponding value for Urvashi is 20 days. How many days it would
have taken for the work to complete had both ladies not done overtime?

(1) 20 (2) 23 (3) 30 (4) can not be determined (5) none of


these

Solution:

Let the number of days required by yana to finish the work alone be x days of 8 hrs.
Let the number of days required by urvashi to finish the work alone be y days of 8 hrs.
Let the number of days of overtime by yana and urvashi be p and k respectively.
So,in 12 days yana finishes 12/x+k/2x of work.
In 12 days,urvashi finishes, 12/y+p/2y of work.
So by condition 1, 12/x+12/y+k/2x+p/2y=1....(1)
By condition 2, 15/x+15/y+p/2y=1.....(2)
And,by condition 3, 20/x+20/y+k/2x=1.....(3)
Adding 2 and 3 and subtracting 1 from the result,
1/x+1/y=1/23.

So,if both work normally,they take 23 days.

Ö Choice (2) is the right answer

94. An Elective on strategy is attended by 15 students sitting on adjacent chairs in a


circle. The Prof. enters the class with 6 copies of material on BSC. The prof. wishes
to distribute the copies in such a manner that each student gets at most 1 copy and
anyone who hasn't should be able to read from his immediate (left or right)
neighbour's copy. In how many ways can the copies be distributed?

(1) 60 (2) 75 (3) 90 (4) 125 (5) 150

Solution:

Suppose that you are one of the student (X); then there’s a 6/15 chance that you’ll get one of the
copies. Given that you do get a copy, how many ways are there to distribute the rest? We need
only multiply the answer to that question by 15/6 = 5/2 to answer the original question.

Going clockwise around the circle from student (X), write down the sizes of the gaps between
students with copies. There are six such gaps, each of size 0–2, and the sum of their sizes must be
15 − 6 = 9. So the gap sizes are either 1, 1, 1, 2, 2, 2 in some order, or 0, 1, 2, 2, 2, 2 in some order.
Remember, the copies are indistinguishable.

In the former case, 6!/3!3! = 20 orders are possible; In the latter, 6!/1!1!4! = 30. => there are 20
+ 30 = 50 possibilities.

Multiplying this by 5/2, gives 125

=> Choice (4) is the right answer

95. Let x= p/11 satisfies log(2x-3/4)/log x > 2, where p is an integer. Then the
number of possible p is
(1) 2 (2) 6 (3) 3 (4) 5 (5) none of these

Solution:

(log(2x-3/4) – 2logx).log x > 0 solving


Case 1: When x > 1 => (2x-3/4) > x^2.
Case 2: When x < 1 => (2x-3/4) < x^2 && x > 3/8
=> x lies in (3/8, ½) U (1, 3/2)

=> Choice (2) is the right answer

96. How many natural numbers n are there such that out of all the positive divisors
of number n (other than both 1 and n) the largest one is 15 times than the smallest
one?

(1) 1 (2) 2 (3) 3 (4) There are no such numbers (5) Infinitely
many

Solution:

(If S is the smallest asked factor and L the largest then S.L = N => 15S^2 = N. But S can take
values less than or equal to the smallest prime factor of 15 => S = 2, 3.

=> Choice (2) is the right answer

97. F is a fixed point in the plane. P, Q, R are points such that FP = 3, FQ = 5, FR = 7


and the area PQR is as large as possible. Then F must be (of PQR)

(1) incentre (2) orthocentre (3) circumcentre (4) centeroid (5) none

Solution:

Consider all points A' such that FA' = 3. They lie on a circle center F. The area of A'BC is BC/2
times the distance of A' from BC. That distance is maximal for A'F perpendicular to BC (because
the distance is the distance of P from BC + FA'sin x, where x is the angle between A'F and BC).
Hence AF must be perpendicular to BC. Similarly BF must be perpendicular to AC, so F must be
the orthocenter.

=> Choice (2) is the right answer

98. Consider two distinct positive integers x and y having integer arithmetic,
geometric and harmonic means. The minimum value of |x - y| is

(1) 15 (2) 20 (3) 30 (4) 40 (5) 50

Solution:

Clearly both m and n must have the same parity (i.e. either both are odd or both are even). Let m
= a+b, and n=a-b for integers a, b where a > b > 0.
we have |m-n| = 2a, mn = a^2 - b^2 = p^2 is a perfect square. Thus, (p, b, a) form a pythagorean
triplet.
(1/2m + 1/2n)^-1 = (a^2 - b^2)/a => a divides both b^2 and p^2. From this we get the triplet
(20, 15, 25).
=> m = 40, n = 10.
=> Choice (3) is the right answer

99. For how many pair of primes (p, q) does there exist natural number n such that
(p^2+1).(q^2+1) = n^2+1?
(1) 2 (2) 3 (3) 5 (4) 6 (5) none of these

Solution:

Case 1: when each of p, q > 3 => p and q are of the form 6k+/-1.
We need to find (p, q) such that p^2 + q^2 + p^2.q^2 is a perfect square. Each of p^2, q^2,
p^2.q^2 is of the form 4t+1 => n^2 is of the form 4r+3, which is a contradiction as perfect
squares are of the form 4r+1 or 4r.

Case 2: when p = 3, or p = 2, and q is any prime -> Solve this yourself.

Alternate Solution:

If 'p' and 'q' are both odd,


(p^2+1)(q^2+1) = 4x = n^2+1
which implies that n^2 = 4x-1.
Since the squares are of form 4x or 4x+1, 'p' and 'q' are not both odd. Hence, one of the prime
numbers in solution, if any, is 2.

Say, p = 2
This gives us, 5(q^2+1) = n^2+1 which can be simplified to
5q^2 = (n+2)(n-2)
5q^2 can be factorized in two factors as (1,5q^2), (q,5q) or (5,q^2).
For (1,5q^2) n=3 which means q=1, hence not possible. This also removes the set (q,5q).
For (5,q^2) n=7 (as n=3 has already been ruled out). This gives q=3.

The only feasible pair (p, q) is (2, 3) and (3, 2)

=> Choice (1) is the right answer

100. The average value of |a - b| + |c - d| + |e - f| for all possible permutations a, b, c ,


d, e, f of 1, 3, 5, 7, 9, 11 is

(1) 21 (2) 18 (3) 12 (4) 14 (5) none of these

Solution:

By symmetry the average of |a - b| is independent of choices in which they are chosen (provided
they are unequal). Suppose a = k. Then the average of |k - b| is (k-1 + k-3 + ... + 2 + 2 + 4 + ... +
11-k)/5 = 2*(k^2 -7k + 21)/5. So average of |a - b| is (1/6)*2*sigma (k^2 -7k + 21)/5. Hence
required average is 2*sigma(k^2 -7k + 21)/10 = 2*(91 - 147 + 126)/10 = 14.

=> Choice (4) is the right answer

101. A gathering of a certain number of families consists of people belonging to two


generations only. It is known that the number of families is less than the number of
girls, the number of girls is less than the number of boys and that the
number of boys is less than the number of parents. If the minimum number of
single parent families is two, then what is the minimum number of families, given
that no family has more than 3 children?
(1) 3 (2) 4 (3) 5 (4) 7 (5) none of these

Solution:

Let no of families , girls , boys and parents be : F,G,B,P respectively.

Now , F<G<B<P

1) F cant be 2 as minimum number of single parent families is 2 and hence 2 parents.

F<G<B<P
2) 3<4<5<6 : 6 parents not possible with 2 single parent families(2*1) and one family with two
parents (1*2)

3) 4<5<6<7 : 7 parents not possible (2*1 + 2*2)

4) 5<6<7<8 : Possible (P = 2*1 + 3*2)

Hence the minimum no. of families = 5

=> Choice (3) is the right answer

102. Let f(x + f(x)) = x for all real x, and if f(ax + bf(x)) = cx + df(x), then which
among the following is necessarily true?

(1) b = c (2) b = d+1 (3) a = d (4) at least 2 of the foregoing (5) none

Solution:

Substitute x = x + f(x) in f(x + f(x)) = x and we will have our answer. Please refer the discussions
thread for more on this.

=> Choice (1) is the right answer

103. How many equilateral triangles of side 2/√3 are formed by the lines y = k, y =
x(√3) + 2k, y = -x(√3) + 2k for |k| <= 10 where k is an integer?

(1) 600 (2) 660 (3) 720 (4) 780 (5) none of these

Solution:

Between the lines y = 10 and y = -10 we have lines parallel to them, but outside them we do not.
Similarly for the lines y = x√3 ± 20, and for the lines y = -x√3 ± 20. Thus the area where triangles
are formed is the hexagon bounded by these six lines. It has long diagonal length 40/√3 from -
20/√3 (the intersection of y = 0, y = x√3 + 20 and y = -x√3 - 20) to 20/√3 (the intersection of y =
0, y = x√3 - 20 and y = -x√3 + 20). So we can regard it as made up of 6 equilateral triangles side
20/√3. Each of these is divided into equilateral triangles side 2/√3. Each has side 1/10 of the large
triangle, so area 1/100, so there are 100 of them, or 600 in all. But there is a trap. There is a line of
triangles outside each edge of the hexagon (with bases on the hexagon). Each edge has 10
triangles, so 60 in all.

=> Choice (2) is the right answer


104. S is a set of positive integers containing 1 and 99. No elements are larger than
99. For every n in S, the arithmetic mean of the other elements of S is an integer.
What is the largest possible number of elements of S?
(1) 8 (2) 10 (3) 12 (4) 15 (5) none of these

Solution:

Let us tackle this problem with small example.


Let we have 4 numbers in S, and let these be a, b, c, d
Thus, (a+b+c)/3, (b+c+d)/3, (a+c+d)/3 and (a+b+d)/3 are each integers.
=> subtract each term from any other => each of a, b, c, d are equal modulo (3) i.e. each on
division by 3 leave same remainder.
Extending this when we have n numbers with us then each will be equal modulo (n-1).
This was the funda part.

We start the calculation now -> Let we have n numbers [the modulo (n-1) is 1 as the 1st term is 1].
1, 1+(n-1).f, 1+(n-1).2f, .... 1+(n-1).(n-1)f
The last term is 1+(n-1).(n-1)f = 99 => (n-1)^2.f = 98 which yields integer value of n for f = 2. =>
we can at max have 8 numbers in our set.

=> Choice (1) is the right answer

105. For a positive integer i define p(i) as the product of base 4 digits of i e.g. 27 in
base 4 is 123 => p(27) = 1*2*3 = 6. What is the numerical value of p(1) + p(2) + p(3) +
... + p(255)?

(1) 1496 (2) 1554 (3) 1592 (4) 1636 (5) 1684

Solution:

The sum of all p(i) where i has 1 digit in base 4 is (0+1+2+3) = 6


The sum of all p(i) where i has 2 digits in base 4 is (0+1+2+3)(0+1+2+3) = 6^2
The sum of all p(i) where i has 3 digits in base 4 is (0+1+2+3)(0+1+2+3)(0+1+2+3) = 6^3
The sum of all p(i) where i has 4 digits in base 4 is (0+1+2+3)(0+1+2+3)(0+1+2+3)(0+1+2+3) =
6^4

Note that 255 = 3.(1+4+4^2+4^3) is the largest decimal number that can be expressed as 4 digit
number in base 4

=> Choice (2) is the right answer

106. The inscribed circle of an isosceles triangle ABC is tangent to side AB at point D
and bisects the segment CD. If CD = 6√2. Which among the following can not be true
about ABC?

(a) The perimeter is 24


(b) It's obtuse angled
(c) The bisector segment of the smallest angle is 6√2
(d) The perimeter is 28
(e) none of the foregoing

Solution:

Let in triangle ABC the incircle meet AB at D, BC at F and AC at E.


Case 1: AB = AC

Now CE^2 = 6√2*3√2 = 36 => CE = CF = 6,


Also in an isosceles triangle AF will contain the incentre, circumcentre , centroid and orthocentre.
=> AF will bisect BC hence BF = 6 = BD.

In triangle CDB BD = 6 CD = 6√2 and BC = 12,


=> cos(< B) = 3/4 -> FB / AB = 3/4 -> AB =8.
=> the perimeter is 28 and angle A is obtuse (square of the largest side is greater than sum of
square of other 2 sides) so it is an obtuse angled triangle. So (2) and (4) are true here.

Case 2: When AC = BC

we already have CE = BF = 6 and CD is also perpendicular to AB. Let AD = AE = a, applying


pythagorean
theorem for triangle ADC, we get a = 3 => AC = BC = 9, and AB = 6
=> (1) and (3) are also true.

=> Choice (5) is the right answer

Quantitative Question # 107


--------------------------------------------------------
The number of triplets (A, B, C) where A, B, C are subsets of {1, 2, 3} and
A/B/C is NULL, A/B is not NULL, and B/C is not NULL is (A/B denotes intersection
set of A and B)

(1) 12 (2) 18 (3) 27 (4) 36 (5) 48

Solution:
Assume the Veinn-Diagram with 3 subsets A, B and C. As usual Universal set gets
divided into 8 regions. The region A/B/C is NULL. Start from here.
Any element can go in the 7 regions in 7 ways, 3 elements will go in 7^3 ways. Subtract
some cases as given in the question and we will have our answer.

A - Total Ways 7 regions can be filled = 7*7*7 = 343


B - Total ways A/B is null 6*6*6 = 216
C - Total ways B/C is null 6*6*6 = 216
D - Total ways A/B is null and B/C is Null = 5*5*5 = 125
E= Total way A/B or B/C is null = B+C-D = 307
=> our desired answer, A- E = 36
=> Choice (4) is the right answer

Quantitative Question # 108


--------------------------------------------------------
The number of real pairs (p, q) such that whenever α is the root of x^2 + px+q = 0
then α^2 - 2 is also its root is

(1) 2 (2) 4 (3) 6 (4) 3 (5) none of these


Solution:
Case 1: a = b => (a^2-2) = (b^2-2)
=> a = a^2 - 2 => a = 2 or a = -1.
a = b =2 => (p,q) = (-4,4)
a = b = -1 => (p,q) = (1,1)

Case 2: a = (a^2-2) ; b = (b^2-2)


=> (a = 2 or a = -1 )and (b = 2 or b = -1)
=> a = 2, b = -1 or vice versa => (p,q) = (1,-2)

Case 3: a = (b^2-2) ; b = (a^2-2)


=> a = (a^2-2)^2 -2 (substituting the value of b from above)
=>a^4 - 4a^2 - a + 2 = 0
=> (a-2) (a^3 + 2a^2 - 1) = 0
=> (a-2) (a+1) (a^2 + a -1) = 0
a = 2 and a = -1 have already been handled.
a^2 + a - 1 = 0
a = (-1 + root(5))/2 => b = a^2 - 2 = (-1 - root(5))/2
and vice versa.
This gives (p,q) as (1,-1)

Case 3: when a and b are different and a = a^2 - 2 = b^2 - 2


This gives (p, q) as (0, -1) and (0, -4). Try this yourself!

Thus in all we have 6 pair of (p, q) -> (-4, 4), (2, 1), (-1, -2), (1, -1), (0, -4), (0, -1).

Ö Choice (3) is the right answer

Quantitative Question # 109


--------------------------------------------------------
Priyanka does daily work for a fixed time, and her efficiency would reduce by 10%
on each subsequent day. However, when she took a break for a day her efficiency
would come to normal the next day. Assume Priyanka worked for a long time, then
after how many days of consecutive work does she needs to take a break to have the
maximum output?
(1) 3 (2) 4 (3) 5 (4) 7 (5) none of these

Solution:
Let her efficiency be E per day. Then for 2nd day Priyanka's efficiency would be 0.9 E.

Thus, if she were to take break after 2 days of work then Output will be (1+0.9+0)E/3 =
0.633 E. Her efficiency in day 3 without break is 0.81 => if she were to take break after 3
days her output is (1+0.9+0.81+0)E/4 = 0.67 E.

If she were to take break after 4 days her output is (1+0.9+0.81+.729+0)E/5 = 0.68 E.
But, her efficiency without break on 5th day is .6561 E => from here on the average
output above can only go down.
Ö Choice (2) is the right answer

Quantitative Question # 110


--------------------------------------------------------
12 students are sitting on chairs, one on each at most, in 3X5 array i.e. 3 rows and 5
columns, such that the top and bottom rows are all filled and middle row chairs are
empty alternatively. The teacher wishes to re-assign the seats such that every
student moves by 1 seat either left, right, forward or backward e.g. for student at
seat (1, 1) , only 2 choices are possible and for student at seat (2, 5) 3 choices are
possible. In how many ways is this re-assignment possible?

(1) 0 (2) 12C3 (3) 15C3 (4) 12P3 (5) none


of these

Solution:
Color each chair alternately Black and Red => We have 8 Black and 7 Red chairs
and 8 Black and 4 Red are occupied. In the re-assignment every black should come
to Red and every Red should come to black chair => our final configuration should
have 8 Red occupied, but only 7 are available!
Ö Choice (1) is the right answer

Quantitative Question # 111


--------------------------------------------------------
Spiderman has to chase a gang of robbers riding on a car after looting a nearby
bank . Spiderman is having coffee at a distance 36 kms south of the car initially
when his spider senses give him an inkling.So he heads for the car immediately at a
velocity of 50 km/hr as soon as the car also start at a velocity of 40 km/hr. The
gangsters drive the car towards the east with 40 km/hr . Spiderman keeps moving
towards the car following his inkling. (i.e. the direction of Spiderman keeps
changing and he keeps following the shortest path to the car.) The time in hours
after which Spiderman reaches upto car and gives gangster the treatment is

(1) 1.5 (2) 2 (3) 2.5 (4) 3 (5) none of these

Solution:
Let at some point of time spiderman is following the car with his velocity making
angle a with east direction...

Now horizontal velocity of spider man = vcos(a)


The integrals below used are definite from 0 to t.
Horizontal distance covered by spiderman= ∫vcos(a)dt= v*∫cos(a)dt = Horizontal
distance covered by robbers= 40t
So ∫cos(a)dt= 4t/5

Spiderman approaches robbers at a relative speed of 50-40cos(a)…so in time t this


distance = 36
∫(50-40cos(a))dt = 36 => 50t- 40∫cosadt =36
Putting value of cos(a)dt
50t-40*4t/5 = 36 =>18t=36 or t= 2hrs
Ö Choice (2) is the right answer

Solution to Quantitative Question # 112


--------------------------------------------------------
I) A committee has met 40 times, with 10 members at every meeting. No two people
have met more than once at committee meetings => There are more than 60 people
on the committee.
II) One cannot make more than 30 subcommittees of 5 members from a committee
of 25 members with no two subcommittees having more than one common member.

Which of the above is not true?

(1) only I (2) only II (3) I && II (4) none of the foregoing

Solution:
I) Each meeting involves 10.9/2 = 45 pairs. So after 40 meetings, there have been
1800 pairs. We are told that these are all distinct. But if there are N people on the
committee, then there are only N(N-1)/2 pairs available. For N=60, this is only 1770.
=> I is true
II) A subcommittee of 5 has 5.4/2 = 10 pairs. So 31 subcommittees have 310 pairs,
and these are all distinct, since no two people are on more than one subcommittee.
But a committee of 25 only has 25.24/2 = 300 pairs available.
=> II is true

Ö Choice (4) is the right answer

Quantitative Question # 113


--------------------------------------------------------
How many solutions does n^2 - [n^2] = (n - [n])^2 have satisfying 1 ≤ n ≤ 4 where
[n] denotes greatest integer less than or equal to n?

(1) 7 (2) 9 (3) 13 (4) 17 (5) none of these


Solution:
Put n = a + k, where a = [n], and 0 ≤ k < 1. Then the equation is 2ak = [2ak + k^2].
The rhs is an integer, so 2ak must be an integer, and that is evidently a sufficient
condition. So k = 0, 1/2a, 2/2a, ... , (2a-1)/2a. In other words, there are 2 solutions for
a = 1 (namely 1 and 1 1/2), 4 solutions for a = 2 (namely 2, 2 1/4, 2 1/2, 2 3/4), 6
solutions for a = 3, ... , 2(m-1) solutions for a = m-1. Hence there are 2(1 + 2 + ... + m-
1) + 1 = m(m-1) + 1 solutions in the range 1 ≤ x ≤ m.
Ö Choice (3) is the right answer

Quantitative Question # 114


--------------------------------------------------------
Sanjog and Rahul took part in a two-day maths contest. At the end both had
attempted questions worth 500 marks. Sanjog scored 160 out of 300 attempted on
the first day and 140 out of 200 attempted on the second day, so his two-day success
ratio was 3/5. Rahul's attempted figures were different from Sanjog's (but with the
same two-day total). Rahul had a positive integer marks on each day. For each day
Rahul's success ratio was less than Sanjog's. What is the largest possible two-day
success ratio that Rahul could have achieved?

(1) 81/125 (2) 349/500 (3) 131/250 (4) 7/25 (5)


none of these

Solution:
Sanjog's ratio was 8/15 on day 1, 7/10 on day 2, and 8/15 < 7/10. We want Rahul's
ratio to be weighted towards day 2, so we take 1/2 on day 1 and 348/498 on day 2
(we must have a total of 500 attempted), giving 349/500 overall. .
Ö Choice (2) is the right answer

Quantitative Question # 115


--------------------------------------------------------
How many ordered pairs of positive integers m, n satisfy m <= 2n <= 50, n <= 2m <=
50?
(1) 313 (2) 325 (3) 337 (4) 349 (5) none of these

Solution:
There are 625 points (m,n) such that 1 <= m,n <= 25. Out of these 2m < n and 2n <
m have to be excluded.
The number of points satisfying 2n < m is 0 for m=1,2; 1 for m=3,4;...;11 for m=23,
24; 12 for m=25
=> total points are 2(1+2+3+ ... + 11) + 12 = 144
By symmetry 2m < n also gives 144 points.
=> in all 625 - 2*144 = 337 pairs satisfy m <= 2n <= 50, n <= 2m <= 50
Also, refer the post #590 in the discussions thread by dewan.
Ö Choice (3) is the right answer
Quantitative Question # 116
--------------------------------------------------------
There are three cards each with a different positive integer. In each round of a game
between 3 players, the cards are randomly dealt to the players and each receives the
number of counters on his card. After two or more rounds, Abhinav has received
20, Sushil 10 and Vijender 9 counters. In the last round Sushil received the largest
number of counters. Who received the middle number in the first round?

(1) Abhinav (2) Sushil (3) Vijender (4) Abhinav or Sushil (5) Sushil or
Vijender

Solution:
The total of the scores, 39, must equal the number of rounds times the total of the
cards. But 39 has no factors except 1, 3, 13 and 39, the total of the cards must be at
least 1 + 2 + 3 = 6, and the number of rounds is at least 2. Hence there were 3 rounds
and the cards total 13.
The highest score was 20, so the highest card is at least 7. The score of 10 included at
least one highest card, so the highest card is at most 8. The lowest card is at most 2,
because if it was higher then the highest card would be at most 13 - 3 - 4 = 6,
whereas we know it is at least 7. Thus the possibilities for the cards are: 2, 3, 8; 2, 4,
7; 1, 4, 8; 1, 5, 7. But the only one of these that allows a score of 20 is 1, 4, 8. Thus the
scores were made up: 8 + 8 + 4 = 20, 8 + 1 + 1 = 10, 4 + 4 + 1 = 9. The last round
must have been 4 to Abhinav, 8 to Sushil and 1 to Vijender. Hence, on each of the
other two rounds the cards must have been 8 to Abhinav, 1 to Sushil and 4 to
Vijender.

Ö Choice (3) is the right answer

Quantitative Question # 117


--------------------------------------------------------
If S is a set of positive integers <= 100 with no two distinct elements of S summing to
an element of {7, 12, 33, 45, 69, 81} then the maximum number of elements S can
have is

(1) 50 (2) 60 (3) 68 (4) 72 (5) none of these

Solution:

The key number here is 81. 81 can be expressed as sum of 2 positive integers in 40
ways i.e. 1+80 = 2+79 = 3+78 = .... if we remove each of 1, 2, 3, ..., 40 from the set of
first 100 natural numbers then we have S whose no 2 elements add to any element in
the set {7, 12, 33, 45, 69, 81}.
Ö Choice (2) is the right answer
Quantitative Question # 118
--------------------------------------------------------
In a triangle ABC, right angled at B, the median through C bisects the angle
between CA and the bisector of < C. Which among the following best describes the
range of AC/CB?

(1) ((√5 -1)/2, 2) (2) (2, 5/2) (3) (√2+1, 2√3) (4) (5/2, 3) (5) (√5, 4)g

Solution:
Let E be the mid-point of AB such that AE = EB = c/2.
CD bisects < C => BD = ac/(a+b). Since, CE bisects < ACD, CD/CA = DE/EA.

CD^2 = a^2 + a^2c^2/(a+b)^2, DE^2 = (c/2 - ac/(a+b))^2


Simplifying, we get a^2{(a+b)^2 + c^2} = b^2(b-a)^2. Using b^2 = a^2 + c^2 and
eliminating c we get b^3 - 2ab^2 -a^2b - 2a^3 = 0. Put p = b/a, we have p^3 - 2p^2 -
p - 2 = 0.

Let f(p) = p^3 - 2p^2 - p - 2. The 1st derivative of f(p) is 3p^2 - 4p - 1 > 0 for p > 2
and thus a increasing function for p > 2. Also f(2).f(3) < 0 => p lies in (2, 3).

Looking at the options, we have to find the narrowest interval which fits the range
of p.
f(2.5) < 0, and thus this works. p = b/a lies in (5/2, 3).

Ö Choice (4) is the right answer

Quantitative Question # 119


--------------------------------------------------------
n+2 students played a tournament. Each pair played each other once. A student
scored 1 for a win, 1/2 for a draw and nil for a loss. Two students scored a total of 8
and the other students all had equal total scores. How many values of n are
possible?

(1) 0 (2) 1 (3) 2 (4) more than 3 (5) none of these

Solution:
There are (n+2)(n+1)/2 matches, so the total score is (n+2)(n+1)/2. Let the other
students score k each. Then 8 + nk = (n+2)(n+1)/2, so n^2 - (2k-3) - 14 = 0. We know
this equation has one root which is a positive integer. The product of the
roots is -14, so the possibilities for the roots are: 1, -14; 2, -7; 7, -2; 14, -1. Hence the
sum of the roots is -13, -5, 5, or 13 (respectively). Hence k = -5, -1, 4 or 8
(respectively). But k must be non-negative, so n = 7 or 14 is a necessary condition.
We need to check that these values can be achieved. Take n = 7, so there are 9
students in total. If every match is a draw, then every student draws 8 matches and
scores 4, which satisfies the conditions. Take n = 14, so there are 16 students in
total. Suppose one student loses to everyone, and all the other games end in a draw.
Then the first student scores 0 and all the other students score 1 + 14/2 = 8. That
also satisfies the conditions
Ö Choice (3) is the right answer

Quantitative Question # 120


--------------------------------------------------------
The area enclosed by the graph of |x - 60| + |y| = |x/4| is

(1) 120 (2) 240 (3) 360 (4) 480 (5) none of these

Solution:

Case 1: when x>=60


a) And y>0 3x+4y=240
b) And y<0 3x-4y=240

Case 2: when 0=<x<60


a) And y>0 5x-4y=240
b) And y<0 5x+4y=240

Case 3: when x<0


a) And y>0 3x-4y=240
b) And y<0 3x+4y=240

Points of intersection
(48,0), (60,15), (80,0),(60,-15)
The figure is a kite. The area is 1/2*(product of the diagonals).
Ö Choice (4) is the right answer

Quantitative Question # 121


--------------------------------------------------------
Let p, q be non-zero integers. Then minimum possible value of |5p^2 +11pq - 5q^2|
is

(1) 2 (2) 3 (3) 4 (4) 1 (5) none of these

Solution:
Let's see this choice by choice. The min value of 4 is possible iff both p and q are
even integers => if 4 is possible for some choice (m, n) of (p, q) then min 1 is possible
for some choice (m/2, n/2) of (p, q). The min 2 is possible iff p and q are both even
but then 5p^2 + 11pq -5q^2 will be div by 4 => we need to check for 1, 3.
Students are advised to prove why 5p^2 + 11pq -5q^2 = 1 and 5p^2 + 11pq -5q^2 =
3 is not possible for non-zero integers (p, q).

No need to find the actual value [which infact is 5 for p = 2, and q = 5]

Ö Choice (5) is the right answer

Quantitative Question # 122


--------------------------------------------------------
Let f(x) be a polynomial with integral coefficients such that f(x) = f(-x) > 0 for all x >
0. If f(5) = 6, and f(x) is distinct for all X > 0. Then minimum possible value of f(6)
will be

(1) 7 (2) 9 (3) 13 (4) 17 (5) none of these

Solution:
Here also like last QQAD, we solve with eliminating the options

(1) f(x) is an even polynomial => it will only have even powers of x.
(2) f(5) = 6 => f(x) = 6 + (x^2 - 25).g(x) where g(x) is an even function
(3) f(x) > 0 for all x -> by putting g(x) = x^2 - 23, we have f(x) that satisfies
conditions 1, 2, 3.
(4) f(x) is distinct for all x > 0.

g(x) will have its co-efficients as integers => f(6) = 6 + 11.g(6) can't be each 7, 9, 13 as
g(6) is an integer. If g(6) = 1 , not all of g(1), g(2), g(3), g(4) can be < 1 for f(x) > 0 for
all x > 0

Ö Choice (5) is the right answer

Quantitative Question # 124


--------------------------------------------------------
A point (x, y) is called lattice iff both x and y are integers. How many lattice points
are inside the quadrilateral whose four sides are on the lines x = 100, x = 300, y = x/3
+ 0.1 and y = x/3 + 0.6?

(1) 67 (2) 60 (3) 50 (4) 100 (5) none of these

Solution:
Points will lie inside the quadrilateral:
If 100< x < 300
And x/3+.1 < y < x/3 +.6
for x=100, 33.43<y<33.93
as the 2 lines are parallel, we can have at most 1 lattice point for every x in
(100,300). So we need to find for what x's y will be an integer between (x/3 +.1, x/3
+.6). : all x of kind 3x+2 will give such lattices.
First one is 101, and last one is 299.
so 299=101 + (n-1)*3 => n = 67.

Ö Choice (1) is the right answer

Quantitative Question # 125


------------------------------------------------------

Let a(1), a(2), a(3), ..., a(n) be a sequence of integers such that -1 <= a(i) <= 2 for all i
= 1, 2, 3, ..., n. It is given that a(1) + a(2) + a(3) + ... + a(n) = 19, and (a(1))^2 +
(a(2))^2 + (a(3))^2 + ... + (a(n))^2 = 99.
Let m and M be the minimum and maximum possible values of (a(1))^3 + (a(2))^3 +
(a(3))^3 + ... + (a(n))^3 respectively, then M/m equals

(1) 3 (2) 4 (3) 7 (4) 9 (5) none of these

Let number of 2s be = a, let number of 1s be = b, let number of -1s be = c

Thus,
2a + b - c = 19
4a + b + c = 99

Adding this gives 3a + b = 59

Now we have to find maximum and minimum value of P = 8a + b - c

When b = 2 a = 19 M = 133 max


When b = 59 a = 0 M = 19 min

so the maximum(P)/minimum(P)
answer is 133/19 = 7

Ö choice (3) is the right answer

Quantitative Question # 126


--------------------------------------------------------
A village has n residents, named P(1), P(2), • • • , P(n). Each either tells the truth or
lies all the time. For each k: If k is a perfect square, P(k) says that P(k+1) is lying.
Otherwise, P(k) says that P(k+1) is telling the truth. (P(n) talks about P1.) What is
the minimum number of residents, given that n > 100?
(1) 109 (2) 110 (3) 111 (4) 121 (5) none of these

Solution:
The problem has 2 cases : Is "1" the first perfect square lying or tell the truth?

Case 1: 1 tells the truth


If 1 is truthful then 2,3,4 are lying. Since 4 lies by saying that 5 is lying, 5 is telling
the truth.
Which means 5,6,7,8,9 tell the truth but 10 lies and so on. Even perfect squares lie.
So the following tell the truth -> 1,9,25,49,81,121,...
If 121 is the last person, then according to 121, P(1) lies. But 1 is truthful according
to the assumption. So option 4 is ruled out.
100 is lying so 101 is truthful. if 101 were the last person, 1 would be truthful
too(which 1 is), hence 101 as the answer can be deduced without case 2.

Case 2: 1 lies
if 1 is lying then 2,3,4 are truthful. Which means 5,6,7,8,9 lie. But 10 is truthful and
so on. Even perfect squares tell the truth. Hence 4,16,36,64,100,.. tell the truth.
100 truthfully says 101 is lying. If 101 was the last person, then he lies that 1 is
truthful. Since 1 isn't truthful, 101 is lying and all conditions are satisfied.
Hence minimum number of people required (for n>100) is 101.

Ö Choice (5) is the right answer

Quantitative Question # 127


--------------------------------------------------------
CAT 200X quant had two sections, each with at least one question and 28 questions
in total. Each student in a certain class attempted 7 questions. Each pair of
questions was attempted by just two students. Which among the following is true?

(1) Each question was attempted by 9 students


(2) One student attempted either nil or at least 4 questions in the first section
(3) There were 36 students in the class
(4) All of the above
(5) Exactly two of the above

Solution:
Each student attempts 7 questions and hence 21 pairs of questions. There are
28•27/2 = 378 pairs of questions in total and each is attempted by 2 students. So
there must be 378•2/21 = 36 students. Suppose n students solved question 1. Each
solved 6 pairs involving question 1, so there must be 3n pairs involving question 1.
But there are 27 pairs involving question 1, so n = 9. The same applies to any other
question. So each question was solved by 9 students => (1) and (3) are true.
Suppose (2) is false. Suppose there are m questions in the first section, that the
number of students solving 1, 2, 3 questions in the first section is a, b, c respectively.
So a + b + c = 36, a + 2b + 3c = 9m. Now consider pairs of problems in the first
section. There are m(m-1)/2 such pairs. Students solving just 1 solve no pairs, those
solving 2 solve 1 pair and those solving 3 solve 3 pairs, so we have b + 3c = m(m-1).
Solving for b we get b = - 2m^2 + 29m - 108 = -2(m - 29/4)^2 - 23/8 < 0.
Contradiction. So the result must be true.

Ö Choice (4) is the right answer

Quantitative Question # 128


--------------------------------------------------------
Implex purchased Honda Civic recently, but the faulty car odometer of Civic
proceeds from digit 3 to digit 5, always skipping the digit 4, regardless of position. If
the odometer now reads 002005 (starting with 000000), how many km has Civic
actually travelled?

(1) 1284 (2) 1462 (3) 1542 (4) 1604 (5) none of these

Solution:
By skipping a digit we are taking in base 9, and 2005 in base 9 is 2003 + 1 in base 10
= 2*9^3 + 3 + 1 = 1462.
Ö Choice (2) is the right answer

Quantitative Question # 129


--------------------------------------------------------
Sara, Kyna, Riddhi can complete the work W1, W2, W3 alone in 6, 9 and 15 days
respectively. If (Kyna, Riddhi), (Riddhi, Sara) and (Sara, Kyna) can do the work
W1, W2, W3 respectively in n days each, then n lies in

(1) (3, 3.5) (2) (3.5, 4) (3) (4, 4.5) (4) (4.5, 5) (5) either none or
atleast 2 of these

Solution:
If the amount of work done per day by Sara, Kyna, Riddhi per day are S/6, K/9,
R/15 then K/9 + R/15 = S/n, R/15 + S/6 = K/n and S/6 + K/9 = R/n. Eliminating S, K
and R we get n^3 + 15n^2 = 405. How? Can you use determinants here and get this
in just 1 step?

Let f(n) = n^3 + 15n^2 - 405 -> and is an increasing function for n > 0, f(4.5) < 0 and
f(5) > 0.

Ö Choice (4) is the right answer


Quantitative Question # 130
--------------------------------------------------------
Let [x] denotes the greatest integer that is less than or equal to x, e.g. [5.43] = 5.Let x
be any number selected randomly such that [√x] = 10. The probability that [√(100x)]
= 100 is

(1) 1/7 (2) 1 (3) 67/700 (4) 1/2 (5) none of the foregoing

Solution:
Since, [x^1/2] = 10 =>
100 <= x = 121 => 10000 <= 100x < 12100
Also,
[(100x)^1/2] = 100 => 10000 <= 100x < 10201
Probability = (10201 - 10000)/(12100 - 10000) = 201/2100 = 67/700

Ö Choice (3) is the right answer

Quantitative Question # 131


--------------------------------------------------------
Akshita is standing at the midpoint of the points where her two other friends
Simran and Aahna are standing. They start moving simultaneously with constant
speeds along the line on which they are standing and they pass through a common
point X after 10, 30 and 40 minutes. If they all meet simultaneously after time T
from the start, then T in minutes is

(1) 60 (2) 70 (3) 90 (4) 100 (5) none of these

Solution:
Let distance between Simran and Akshita be d and speeds of Ahana be x,that of
Akshita be y and of Simran be z.

Given, y/2=d+x/6....(1) and 2z/3=2d+x/6....(2)

That they all meet after time T gives us the following equations

d/(z-y)= T...(3), d/(y-x)=T...(4), 2d/(z-x)=T....(5)

Comparing (3) and (4) we get, z+x=2y....(6)


now from (1) we get, d= (3y-x)/6 ...(7)
substituting in (2) we get, 2y=3x.
thus we get d=7y/18.
T= 7y/18 * 3/y= 7/6hrs = 70mins.

Ö Choice (2) is the right answer


Quantitative Question # 132
--------------------------------------------------------
Given a triangle area of area 6 and perimeter 12, let S be the set of all points a
distance 5 or less from a point of the triangle. What is the area of S to the nearest
integer?

(1) 126 (2) 134 (3) 145 (4) 157 (5) 168

Solution:
The region consists of 3 rectangles outside the triangle who one side is 5 and the
other side as the length of the side of the triangle. Also the region consists of 3
sectors of radius 5 each around the three vertices.
Let ABC be the triangle. Let the rectangles be ABEF, ACFG and CBJI. Then <JBE
+ <GCI + < HAF = 360 degrees and the three sectors complete a circle.

Hence, Area(s) = 6 + 5*12 + 25*pi =~ 145

Ö Choice (3) is the right answer

Quantitative Question # 133


--------------------------------------------------------
Let a fruitseller sells an apple, an orange, a mango and a peach at loss of 10%, loss
of 40%, profit of 20% and profit of 50% respectively. Let the loss made on an apple
and an orange together be as much as in amount the profit made on a mango and a
peach together. Suppose you have the money to buy 2 apples and 12 oranges but
instead you buy 3 mangoes. How many peaches can you buy with rest of the money?

(1) 3 (2) 5 (3) 6 (4) 8 (5) none of these

Solution:
Short cut by Fuzon
Let the initial costs of Apple|Orange|Mango|Peach be 10|10|10|6
The costs at which he sells Appl|orange|Mango|Peach = 9|6|12|9

He has money to buy 2 apples and 12 oranges = 2*9 + 12 *6 = 90


He buys 3 mangoes instead : 12 * 3 = 36 - Left with 54 Rs

He can buy 54/9 = 6 Peaches with that amount !!

Conventional

Apple - CP = a , SP = 0.9a
Orange - CP = r, SP = 0.6r
Mango - CP = m, SP = 1.2m
Peach - CP = p, SP = 1.5p

0.1a + 0.4r = 0.2m + 1.5p => a+4r = 2m + 5p --(1)

Money left
= 2(0.9a) + 12(0.6r) - 3(1.2m)
= 1.8(a+4r) - 3.6m
= 1.8(2m +5p) - 3.6m = 9p = 6 (1.5p) => 6 peaches.
Ö Choice (3) is the right answer

Quantitative Question # 134


--------------------------------------------------------

The 150 Quant devils of QQAD are given individual numbers from 1 to 150, and a
contest happens in multiple rounds to select the Ultimate QQAD devil. The
elimination follows a weirdo pattern. In the 1st round starting from first devil, every
3rd devil is eliminated i.e. 1st, 4th, 7th, .... This repeats again from the first
numbered (among the remaining) devil in the next round (leaving 3, 5, 8, 9, ...). This
process is carried out repeatedly until there is only the winner left. What is the
number of the Ultimate QQAD Devil?

(a) 93 (b) 48 (c) 119 (d) 38 (e) 140

Solution:
Suppose a number is at nth position. First time n/3, [n/3] + 1, [n/3] + 1 integers less
than n are eliminated for n = 3k, 3k+1, 3k+2. So, nth number's new position will be
2n/3, eliminated, [2n/3] for n = 3k, 3k+1, 3k+2.
Working this backwards it's like if a number is at nth position now, earlier it must
have been at 3n/2 or [3n/2] + 1. So, what is at 1st position now (winner) was earlier
at position [3/2] + 1 = 2. Moving back one more step what was at position 2, even
earlier was at position 3*2/2 = 3.
Going on like this the only positions that can end up at first position are 1, 2, 3, 5, 8,
12, 18, 27, 41, 62, 93, 140, 210, 315...(These are positions, not actual values)
And when I have moved back enough steps to get back the original series
(1,2,3,4,5,...), positions become equal to actual values cause in the actual series nth
number is at nth position. So these are the actual winning contestant numbers 1, 2,
3, 5, 8, 12, 18, 27, 41, 62, 93, 140, 210, 315... subject to how many total contestants
are there.

Ö Choice (5) is the right answer


Quantitative Question # 135
--------------------------------------------------------
The number of integral k such that log2 + log(2x^2 + 2x + 7/2) >= log(kx^2 + k),
possesses at least one real solution is

(a) 3 (b) 4 (c) 7 (d) 8 (e) none of these

Solution:
log (4x^2 + 4x + 7) >= log(kx^2 + k)
=> for 4x^2 + 4x + 7 >= kx^2 + k, we must have atleast one real solution for k > 0.
=> (k-4)x^2 -4x + (k-7) <= 0

Multiply both the sides by (k-4).

=> LHS of the inequality becomes ((k-4)x - 2)^2 + (k-8 )(k-3).

For k >=4 we have ((k-4)x - 2)^2 + (k-8 )(k-3) <= 0 ... we have k = 4, 5, 6, 7, 8
For k < 4 we have ((k-4)x - 2)^2 + (k-8 )(k-3) >= 0 ... we have k = 1, 2, 3

Ö Choice (4) is the right answer

Quantitative Question # 136


--------------------------------------------------------
There are 10 boxes available, and in each box lies a ball. Nbangalorekar knows that
some of these 10 balls are red, and its number is even. In each query, nbangalorekar
may point to two random boxes and ask if there is a red ball in at least one of these 2
boxes. To which the reply he will get is in a Yes or a No. After how many minimum
number of queries can nbangalorekar indicate two boxes for sure the boxes in which
the red ball lies?

(1) 9 (2) 12 (3) 15 (4) 17 (5) none of these

Solution:
16 queries are not enough.
Suppose the answers to the first 16 questions are all "YES, at least one of these two
boxes contains
a red ball". Suppose furthermore, nbangalorekar in the end claims that boxes 1 and
2 both contain red
balls.

Case 1: If nbangalorekar never queried the pair (1,2), then we make the balls in 1
and 2 non-red, and all other
balls red. This situation agrees with all answers, and nbangalorekar cannot be sure
that he is right.

CASE 2: nbangalorekar has queried (1,2). Then there are 8 pairs (1,x) and 8 pairs
(2,x) with x in
3...10. One of these 16 pairs has never been queried, say the pair (1,z). We make the
balls in box 1
and in box z non-red, and all other balls red. Again, the player cannot be sure that
he is right.

17 queries are enough and here is the alogorithm as to how.


We describe an algorithm with at most 17 queries that correctly identifies two red
balls. The algorithm
goes through two phases.

Phase 1: Query the pairs (1,x) with x=2...10.

CASE 1.A: If one of the answers is NO, then box 1 has a non-red ball. Then the
YES-answers
identify all boxes with a red ball. We stop.
CASE 1.B: If all answers are YES, then the ball in box 1 must be red. (If it was non-
red, then
all other 9 balls are red; contradiction to the even-assumption). We move on to
phase 2.

Phase 2: Query the pairs (2,y) with y=3...10.

CASE 2.A: If one of the answers is NO, then box 2 has a non-red ball. Then the
YES-answers
identify all boxes with a red ball. We stop.
CASE 2.B: If all answers are YES, then the ball in box 2 is red (same argument as in
case 1.B).

We stop with red balls in boxes 1 and 2.

Ö Choice (4) is the right answer

Quantitative Question # 138


--------------------------------------------------------
In the triangle ABC, the length of the altitude from A is not less than BC, and the
length of the altitude from B is not less than AC. Which among the following about
triangle ABC can be true?

(1) obtuse-angled (2) acute-angled isosceles (3) equilateral (4) scalene (5)
none of these
Solution:
Let k be twice the area of the triangle. Then k=BC^2, k=AC^2 and k <= AC.BC,
with equality in the last case only if AC is perpendicular to BC. Hence AC and BC
have equal lengths and are perpendicular. So the angles in degrees are 90, 45, 45.
Ö Choice (5) is the right answer

Quantitative Question # 139


--------------------------------------------------------
What is the volume closest to an integer of a right circular cone that can be formed
from a strip ABC(which is in shape of a right-angled triangle) with C as vertex of
length AB=10 cm and other two sides as 6 cm and 8cm?

(1) 9 (2) 11 (3) 7 (4) 13 (5) none of these

Solution:
A cone can be formed from the sector of a circle.
Since C is the vertex, the angle of sector can be maximum 90 degrees.
The radius of this sector is maximum when its radius is equal to the length of the
prependicular from C to AB which is
6*8/10 = 4.8 cm
=> Slant height of this cone is 4.8 cm and radius of the cone is 1/4*(4.8) = 1.2 cm
Volume of the cone is 1/3*pi*((1.2)^2)*(4.8^2 - 1.2^2)^1/2 =~ 7
Ö Choice (3) is the right answer

Quantitative Question # 140


--------------------------------------------------------
A non-constant polynomial f(x) satisfies 8(x-1)*f(x) = (x-8)*f(2x), then f(x) is the
polynomial of degree

(1) 3 (2) 4 (3) either 3 or 4 (4) 12 (5) can not be determined

Solution:
Setting x = 1, and x = 8 gives us f(2) = 0, f(8) = 0 respectively.
since f(8) = 0, f(4) = 0. If the degree of f(x) is k => f(2x)/f(x) = 8(x-1)/(x-8)
for large x, f(2x)/f(x) -> 8 => 2^k = 8 => k = 3
Ö Choice (1) is the right answer

Quantitative Question # 140


--------------------------------------------------------
The sum of the series 1 + (1+1/2).1/3 + (1+1/2+1/2^2).1/3^2 +
(1+1/2+1/2^2+1/2^3).1/3^3 + .... is
(1) 3/2 (2) 9/5 (3) 2 (4) 5/2 (5) none of these

Solution:
The given series can be re-written as
(1 + 1/3 + 1/3^2 +.....) + 1/2(1/3 + 1/3^2 +.....) + 1/2^2( 1/3^2 + 1/3^3 .....) etc

=> 3/2(1+ 1/6/ 1/36 + ....)

= 9/5
Ö Choice (2) is the right answer

Quantitative Question # 141


--------------------------------------------------------
The triangle ABC has AB and AC unequal. The angle bisector of A meets the
perpendicular bisector of BC at X. The line joining the feet of the perpendiculars
from X to AB and AC meets BC at D. Then BD/DC is

(1) 1 (2) < 1/2 (c) > 1/2, and < 1 (d) > 1 (e) can not be determined

Solution:
Let the perpendiculars from X to the lines AB, AC meet them at Z, Y respectively.
Triangles XBZ, XYC are congruent because XB = XC (X lies on angle bisector), XZ
= XY (X lies on perpendicular bisector) and < BZX = < CYX = 90 degrees.
Hence BZ = CY. Also AZ can be > or < AY (since AB and AC are unequal). By
Ceva's theorem, (AZ/ZB) (BD/DC) (CY/YA) = 1. Hence BD/DC is < 1 or > 1
depending on the ratio AB/AC.
Ö Choice (5) is the right answer

Quantitative Question # 143


--------------------------------------------------------
Let three positive integers p, p^2 + 2, p^3 + 2 be given. Which among the following
is always true?

(1) All the three numbers are prime for atleast 2 values of p
(2) Exactly 2 of these numbers are perfect squares for some p
(3) The product of 2 of these numbers can be expressed as a 6 digit number in the
base of the third number
(4) atleast 2 of the foregoing
(5) none of these

Solution:
If p is prime > 3 then it is always of the form 6k+/-1 => p^2 + 2 is always div by 3 =>
for p = 3 only all the 3 numbers are prime => (1) is false
p^2 < p^2 + 2 < (p+1)^2 => only possibility is that p and p^3 + 2 can be both perfect
squares. If p is a perfect square then it is of the form 4k or 4k+1 => p^3 + 2 is of the
form 4k+2 or 4k+3 => (2) is false

The product (p^2 + 2).(p^3 + 2) has highest power as 5 and hence in base p can be
written as a 6 digit number => (3) is true

=> Choice (3) is the right answer

Quantitative Question # 142


--------------------------------------------------------
Four articles are sold at Rs 50, 60, 70 and 80 fetching profits 20%, 25%, 40% and
50% (may not be in that order). Given that the total cost price of the articles is an
integral number of rupees, the net profit % on the articles together is

(a) 36% (b) 34% (c) 32% (d) 31% (e) none of these

Solution:
If a, b, c, d are the CPs, then the SPs will be: 6a/5, 5b/4, 7c/5, 3d/2
Now, take the possibilities for each value:
6a/5 = 50 => a = 125/3
6a/5 = 60 => a = 50
6a/5 = 70 => a = 175/3
6a/5 = 80 => a = 200/3

Similarily, the values for b,c,d come out to be: (for 50,60,70,80 in that order)
b = 40, 48, 56, 64
c = 250/7, 300/7, 50, 400/7
d = 100/3, 40, 140/3, 160/3

Notice that b is always integral. Also, c cannot be fractional as there is no way we


can account for the /7 factor (while we can for /3 in two cases)
=> c = 50 , 7d/5 = 70
Now, notice that a and d are integral for the same value (60) => none of them will
correspond to this.
=> b = 48, 5b/4 = 60
Now, the combos possible for (a,c) are :
(125/3, 160/3) ; (200/3, 100/3)

case 1: CP = 50 + 48 + 95 = 193
SP = 260
=> profit = 67/193 ~34.5%
case 2: CP = 50 + 48 + 100 = 198
SP = 260
=> profit = 62/198 ~ 31%
=> Choice (5) is the right answer

Quantitative Question # 144


--------------------------------------------------------
There are two squares of side length unity .They are arranged in a way that their
centers coincide .These two squares have their inside area as an octagon whose one
side is 43/99. The area of the octagon is

(a) 1 (b) 7/9 (c) 78/99 (d) 86/99 (e) none of these

Solution:
Let the sides of the triangle be x, y, (x^2+y^2)^1/2 . It is clear that the perimeter of
the triangle is the side of the square 1. Solving, y = (1-2x)/(2-2x) and octagon side =
1-x-y = (2x^2-2x+1)/(2-2x) = 43/99. Hence 198x^2 - 112x + 13 = 0. Product of roots is
13/198. So area octagon = area square - 4 x area triangle = 1 - 2xy = 1 - 26/198 =
86/99.
=> Choice (d) is the right answer

You might also like

pFad - Phonifier reborn

Pfad - The Proxy pFad of © 2024 Garber Painting. All rights reserved.

Note: This service is not intended for secure transactions such as banking, social media, email, or purchasing. Use at your own risk. We assume no liability whatsoever for broken pages.


Alternative Proxies:

Alternative Proxy

pFad Proxy

pFad v3 Proxy

pFad v4 Proxy